[obm-l] Re: [obm-l] Re: [obm-l] Ângulos de um triângulo

2020-12-04 Por tôpico Carlos Victor
 

Use a lei dos senos e o fato de que sen(54º)-sen(18º)=sen(30º). 

Em 04/12/2020 1:50, Anderson Torres escreveu: 

> Em seg., 30 de nov. de 2020 às 19:28, Professor Vanderlei Nemitz 
>  escreveu: 
> 
>> Boa noite!
>> 
>> Alguém conhece uma saída para o seguinte problema? 
>> Muito obrigado! 
>> 
>> NUM TRIÂNGULO ISÓSCELES ABC, AB = AC. 
>> SEJA D UM PONTO INTERNO TAL QUE OS ÂNGULOS DBC, DCB, DBA E DCA MEDEM, 
>> RESPECTIVAMENTE, 12°, 18°, 54° E 48°. 
>> DETERMINE A MEDIDA DO ÂNGULO DAC.
> 
> Eu ainda nao resolvi, mas sei que e 30 graus. 
> 
>> [1]
>> Livre de vírus. www.avast.com [1].

 

Links:
--
[1]
https://www.avast.com/sig-email?utm_medium=emailutm_source=linkutm_campaign=sig-emailutm_content=webmail

Re: [obm-l] Prova interessante de que lim n ---> oo n^(1/n) = 1

2020-10-29 Por tôpico Carlos Victor
 

Muito linda Artur. 

Carlos Victor 

Em 28/10/2020 7:44, Artur Costa Steiner escreveu: 

> Achei essa prova bem imaginativa. 
> 
> Para n>= 2, temos n^(1/n) > 1. n^(1/n) pode ser escrito como 
> 
> n^(1/n) = ((raiz(n) . raiz(n) . 1  1)^(1/n) 
> 
> onde o 1 aparece n - 2 vezes. Logo, n^(1/n) é a média geométrica dos números 
> {raiz(n), raiz(n), 1, . .1}. 
> 
> Pela desigualdade MA >= MG temos, para n>= 2, que 
> 
> 1 < n^(1/n) < (raiz(n) + raiz(n) + 1 +1)/n= (2 raiz(n) + (n - 2))/n 
> 
> 1 < n^(1/n) < 2/raiz(n) + 1 - 2/n 
> 
> Como na desigualdade acima o membro da direita tende a 1 quando n vai para 
> oo, segue-se por confronto que 
> 
> lim n ---> oo n^(1/n) = 1 
> 
> Artur

 

Re: [obm-l] teste

2020-08-08 Por tôpico Carlos Victor
 

Há muito tempo que os meus emails enviados também estão assim e não sei
o motivo. 

Carlos Victor 

PS : este email não sei se chegará aos companheiros da lista 

Em 08/08/2020 17:39, Luís Lopes escreveu: 

> Recebo as mensagens normalmente. Mas não tenho confirmação de 
> chegada ao grupo das que envio. E não aparecem nos arquivos também. 
> 
> Mandei uma há umas 5 horas intitulada polinômio minimal. 
> Chegou? Alguém recebeu ? 
> 
> Luís 
> 
> -- 
> Esta mensagem foi verificada pelo sistema de antivírus e 
> acredita-se estar livre de perigo.

 
-- 
Esta mensagem foi verificada pelo sistema de antiv�rus e
 acredita-se estar livre de perigo.



Re: [obm-l] Dois problemas

2020-04-26 Por tôpico Carlos Victor
 

Para o (1), observar que a_n é periódico e tem período igual a 20, daí
 

Abraços 

Carlos Victor 

Em 26/04/2020 19:21, Rogério Possi Júnior escreveu: 

> Boa noite. 
> 
> Quem pode ajudar com esses dois problemas: 
> 
> 1) (Ibero-1992) Para cada inteiro positivo n, seja a_n o último dígito de 
> 1+2+3+...+n. Calcule a_1+a_2+...+a_n. 
> 
> 2) (UK-1997) N é um número inteiro de 4 dígitos não terminado em zero, e R(N) 
> é o número inteiro de 4 dígitos obtido pela reversão dos dígitos de N; por 
> exemplo R(3275)=5723. Determine todos os inteiros N ára os quais R(N)=4N+3. 
> 
> Sds, 
> 
> Rogério 
> 
> -- 
> Esta mensagem foi verificada pelo sistema de antivírus e 
> acredita-se estar livre de perigo.

 
-- 
Esta mensagem foi verificada pelo sistema de antiv�rus e
 acredita-se estar livre de perigo.



Re: [obm-l]

2020-04-05 Por tôpico Carlos Victor
 

Inscrevendo o triângulo em um círculo, é possível chegar a esta
resposta. 

Carlos Victor 

Em 05/04/2020 19:10, Anderson Torres escreveu: 

> Em dom., 5 de abr. de 2020 às 19:09, Anderson Torres
>  escreveu: 
> Em qui., 13 de fev. de 2020 às 18:19, Vanderlei Nemitz
>  escreveu: 
> Usei várias leis dos senos, obtive coisas legais, mas não o ângulo pedido. 
> Alguém conhece algo interessante?
> 
> Muito obrigado!
> 
> Em um triângulo ABC, em AC localiza-se os pontos consecutivos M,Q e N, tal 
> que AM=NC. Se Q é ponto médio de MN e os ângulos NBC e ABM medem 20º, calcule 
> a medida do ângulo BQC. 
> Se fizermos BAC=\alpha, BCA=\gamma, obtemos que BM/sin alpha = AM/sin
> 20 = CN/sin 20 = BN/sin gamma
> 
> Também, MNB=20+gamma e NMB = 20+alpha.

Dessa forma, usando outra lei dos senos, temos sin alpha / sin
(20+gamma) = sin gamma / sin (20+alpha).

O que nos dá cos (20+2 alpha) = cos (20 + 2 gamma), o que implica
alpha = gamma, ou ABC isósceles, portanto BQC = 90.

>> --
>> Esta mensagem foi verificada pelo sistema de antivírus e
>> acredita-se estar livre de perigo.

 
-- 
Esta mensagem foi verificada pelo sistema de antiv�rus e
 acredita-se estar livre de perigo.



Re: [obm-l] teoria dos numeros

2020-03-30 Por tôpico Carlos Victor
 

Basta fazer (2^3-1)^2n+(2^3+1)^2n -2 e usar binômio de Newton. 

Em 28/03/2020 13:55, Israel Meireles Chrisostomo escreveu: 

> Eu sei resolver o problema abaixo,porém não sei se é a forma mais simples de 
> se fazer.Vcs poderiam por favor colocar suas soluções nos comentários dessa 
> publicação? O problema é o seguinte:
> Prove que 128 divide 49^{n} + 81^{n} -2, para todo n ≥ 1.Se possível não use 
> indução, pois eu já estou usando indução. 
> 
> -- 
> 
> Israel Meireles Chrisostomo 
> -- 
> Esta mensagem foi verificada pelo sistema de antivrus e 
> acredita-se estar livre de perigo.

 
-- 
Esta mensagem foi verificada pelo sistema de antiv�rus e
 acredita-se estar livre de perigo.



[obm-l] Re: [obm-l] Ajuda com dízima

2020-02-24 Por tôpico Carlos Victor
 

Estou conjecturando que 1/3^n tem período igual a 3^(n-2) , para n>=3. 

Carlos Victor 

Em 20/02/2020 18:01, Prof. Douglas Oliveira escreveu: 

> Qual o número de dígitos do período de 1/(3^2005) ? 
> 
> Saudações 
> Douglas Oliveira 
> -- 
> Esta mensagem foi verificada pelo sistema de antivrus e 
> acredita-se estar livre de perigo.

 
-- 
Esta mensagem foi verificada pelo sistema de antiv�rus e
 acredita-se estar livre de perigo.



Re: [obm-l]

2019-11-18 Por tôpico Carlos Victor
 

O Pacini me pediu que enviasse para a lista a ideia abaixo, pois ele não
está conseguindo concluir o devido envio : 

Para n par o link que o Carlos Gustavo colocou mostra a análise. 

Acredito ter encontrado uma outra ideia para todas as soluções com
a=2n+1, usando 

3^(2n+1) = 2(b^2) + 1 

3^(2n+1) = 2(b^2) +3 -2 

3(3^(2n)-1) = 2(b^2 - 1) 

3(3^n-1)(3^n+1) = 2(b-1)(b+1). 

Vou verificar se realmente usando esta ideia chegarei às soluções e
postarei mais adiante. 

Pacini 

Carlos Victor 

Em 16/11/2019 14:47, Pedro José escreveu: 

> Boa tarde! 
> Curioso, a solução (2,2) sai para q =0 no segundo caso 3q+2. 
> Todavia, falta mostrar que para os côngruos de 3 mod81, embora 6q^2+8q+3 
> dívida 81, não é uma potência de 3, já vi que ficou capenga. 
> Saudações, 
> PJMS 
> 
> Em sáb, 16 de nov de 2019 14:54, Pedro José  escreveu: 
> 
>> Boa tarde! 
>> O Esdras conseguiu para a e b par. 
>> Creio ter conseguido para a e b ímpares. 
>> Já havia encontrado (1,1) é (5,11)além de (2,2) para se b pares. 
>> Vamos atrás dos peixes maiores. 
>> 3^a=2*(3q+c)^2+1, 0=> c=1 ou c=2. 
>> Para c=1. 
>> 3^a=18q^2+12q+3 
>> 3^(a-1)=6q^2+4q+1 
>> Note que a solução (1,1) acontece para q =0. 
>> E novamente uma restrição q=2 mod3pois já encontramos a soluçao para q =0. 
>> Mas como não havia soluções menores que as citadas com a e b ímpares, 
>> 3^a>81. 
>> Então 81 |6q^2+4q+1 Para algum resíduo de{5, 8 , 11...77,80}, o que não 
>> acontece. 
>> Para c =2 
>> 3^a =2(3q^2+2)^2+1 
>> 3^(a-1)=6q^2+8q+3 E temos nova restrição q=0 mod3. 
>> Observe que a solução (5,11) vem de q=3. 
>> Usando o mesmo raciocínio anterior, 
>> 81 | 6q^2+8q+3 para algum resíduo de {6,9,12..75 78} 
>> O que não acontece. 
>> Então juntando essa restrição braçal com a refinada do Esdras só existem as 
>> soluções que mencionara lá atrás.: (1,1); (2,2) e (5,11). 
>> Alguém poderia verificar se está correto? 
>> Saudações, 
>> PJMS
> 
> -- 
> Esta mensagem foi verificada pelo sistema de antivrus e 
> acredita-se estar livre de perigo.

 
-- 
Esta mensagem foi verificada pelo sistema de antiv�rus e
 acredita-se estar livre de perigo.



Re: [obm-l]

2019-11-18 Por tôpico Carlos Victor
 

O Pacini me pediu que enviasse para a lista a ideia abaixo, pois ele não
está conseguindo concluir o devido envio : 

Para n par o link que o Carlos Gustavo colocou mostra a análise. 

Acredito ter encontrado uma outra ideia para todas as soluções com
a=2n+1, usando 

3^(2n+1) = 2(b^2) + 1 

3^(2n+1) = 2(b^2) +3 -2 

3(3^(2n)-1) = 2(b^2 - 1) 

3(3^n-1)(3^n+1) = 2(b-1)(b+1). 

Vou verificar se realmente usando esta ideia chegarei às soluções e
postarei mais adiante. 

Pacini 

Carlos Victor 

Em 16/11/2019 14:47, Pedro José escreveu: 

> Boa tarde! 
> Curioso, a solução (2,2) sai para q =0 no segundo caso 3q+2. 
> Todavia, falta mostrar que para os côngruos de 3 mod81, embora 6q^2+8q+3 
> dívida 81, não é uma potência de 3, já vi que ficou capenga. 
> Saudações, 
> PJMS 
> 
> Em sáb, 16 de nov de 2019 14:54, Pedro José  escreveu: 
> 
>> Boa tarde! 
>> O Esdras conseguiu para a e b par. 
>> Creio ter conseguido para a e b ímpares. 
>> Já havia encontrado (1,1) é (5,11)além de (2,2) para se b pares. 
>> Vamos atrás dos peixes maiores. 
>> 3^a=2*(3q+c)^2+1, 0=> c=1 ou c=2. 
>> Para c=1. 
>> 3^a=18q^2+12q+3 
>> 3^(a-1)=6q^2+4q+1 
>> Note que a solução (1,1) acontece para q =0. 
>> E novamente uma restrição q=2 mod3pois já encontramos a soluçao para q =0. 
>> Mas como não havia soluções menores que as citadas com a e b ímpares, 
>> 3^a>81. 
>> Então 81 |6q^2+4q+1 Para algum resíduo de{5, 8 , 11...77,80}, o que não 
>> acontece. 
>> Para c =2 
>> 3^a =2(3q^2+2)^2+1 
>> 3^(a-1)=6q^2+8q+3 E temos nova restrição q=0 mod3. 
>> Observe que a solução (5,11) vem de q=3. 
>> Usando o mesmo raciocínio anterior, 
>> 81 | 6q^2+8q+3 para algum resíduo de {6,9,12..75 78} 
>> O que não acontece. 
>> Então juntando essa restrição braçal com a refinada do Esdras só existem as 
>> soluções que mencionara lá atrás.: (1,1); (2,2) e (5,11). 
>> Alguém poderia verificar se está correto? 
>> Saudações, 
>> PJMS
> 
> -- 
> Esta mensagem foi verificada pelo sistema de antivrus e 
> acredita-se estar livre de perigo.

 
-- 
Esta mensagem foi verificada pelo sistema de antiv�rus e
 acredita-se estar livre de perigo.



Re: [obm-l]

2019-11-18 Por tôpico Carlos Victor
 

O Pacini me pediu que enviasse para a lista a ideia abaixo, pois ele não
está conseguindo concluir o devido envio : 

Para n par o link que o Carlos Gustavo colocou mostra a análise. 

Acredito ter encontrado uma outra ideia para todas as soluções com
a=2n+1, usando 

3^(2n+1) = 2(b^2) + 1 

3^(2n+1) = 2(b^2) +3 -2 

3(3^(2n)-1) = 2(b^2 - 1) 

3(3^n-1)(3^n+1) = 2(b-1)(b+1). 

Vou verificar se realmente usando esta ideia chegarei às soluções e
postarei mais adiante. 

Pacini 

Carlos Victor 

Em 12/11/2019 19:06, Carlos Gustavo Tamm de Araujo Moreira escreveu: 

> Há uma menção a esse problema em 
> https://math.stackexchange.com/questions/2826307/integer-solutions-of-3n-1-2m2
>  [1] 
> Uma sugestão é usar o fato de que Z[i.sqrt(2)] é um domínio de fatoração 
> única, e escrever 1+2b^2 como (1+b.i.sqrt(2))(1-b.i.sqrt(2)). 
> Notem que 3 se fatora aí como (1+i.sqrt(2))(1- i.sqrt(2)). 
> Abraços, 
> Gugu 
> 
> On Tue, Nov 12, 2019 at 7:21 PM Pedro José  wrote: 
> 
> Boa noite! 
> Agora captei vosso pensamento. 
> Só que ao transformar a equação em uma equação de Pell, nós maculamos a 
> função 3^n. 
> Em verdade a solução para a par a= 2n, seria (2,2); pois, como mencionara 
> anteriormente se a é par, b também o é. 
> Só que quando procuramos as outras soluções, baseando-se na propriedade de 
> que a norma em Q [RAiz(A)] conserva a multiplicação. Só que quando eu pego a 
> solução 
> 3 + 2 Raiz(2) e elevo ao quadrado 17 + 12 Raiz(2). Se eu pegar 17^2-2*12^2=1 
> eu atendo x^2 - 2Y^2=1. E assim sucessivamente. Mas não existe n inteiro tal 
> que 3^n=17, então não é uma solução da equação original. 
> Creio que seja um pouco mais complicada a solução. Pois o difícil é saber 
> quando atende também a 3^n. 
> Acredito que deva haver uma forma de restringir a essas soluções, pois, 
> definir em que condições a solução terá x como uma potência de 3 seja bem 
> difícil. 
> Estou apanhando mais do que mala velha em véspera de viagem. 
> Se alguém postar uma solução, me ajudaria bastante. 
> 
> Saudações, 
> PJMS 
> 
> Saudações, 
> PJMS. 
> 
> Em ter., 12 de nov. de 2019 às 17:25, Pedro José  
> escreveu: 
> 
> Boa tarde! 
> Douglas, 
> perdoe-me pela minha miopia, mas você poderia detalhar melhor onde entra a 
> equação de Pell? 
> A equação de Pell não é x^2-Dy^2 = N? 
> Se a é par b é par e se a ímpar b é ímpar para atender mod8, 
> Não consegui captar a sugestão. 
> 
> Saudações, 
> PJMS 
> 
> Em ter., 12 de nov. de 2019 às 16:50, Prof. Douglas Oliveira 
>  escreveu: 
> Hum, então, vamos analisar o caso de a ser par do tipo 2n. 
> 
> Assim podemos escrever que (3^n+b(sqrt2))(3^n-b(sqrt2))=1 
> Dai através da solução mínima que o Pedro fez, como (1,1) por exemplo, da pra 
> ver que são infinitas soluções usando a equação de Pell. 
> 
> Abraco 
> Douglas Oliveira. 
> 
> Em dom, 10 de nov de 2019 19:33, gilberto azevedo  
> escreveu: 
> 
> [HELP] 
> Achas todos os pares (a,b) inteiros positivos tais que : 
> 3^a = 2b² + 1. 
> 
> -- 
> Esta mensagem foi verificada pelo sistema de antivírus e 
> acredita-se estar livre de perigo. 
> -- 
> Esta mensagem foi verificada pelo sistema de antivírus e 
> acredita-se estar livre de perigo.

-- 
Esta mensagem foi verificada pelo sistema de antivírus e 
acredita-se estar livre de perigo. 
-- 
Esta mensagem foi verificada pelo sistema de antivrus e 
acredita-se estar livre de perigo. 
 

Links:
--
[1]
https://math.stackexchange.com/questions/2826307/integer-solutions-of-3n-1-2m2
-- 
Esta mensagem foi verificada pelo sistema de antiv�rus e
 acredita-se estar livre de perigo.



[obm-l] Re: [obm-l] Re: [obm-l] Re: [obm-l] Re: [obm-l] Demonstração com Geometria Plana?

2018-11-24 Por tôpico Carlos Victor
 

Oi Vanderlei, vamos lá: 

Seja ABCD o quadrado de diagonais AC e BD. Sejam os pontos P, E e F como
no enunciado. Tracemos a reta que passa por A e E encontrando o
prolongamento de DC em R.Seja também Q o ponto de interseção da reta que
passa por B e F com o prolongamento de DC.Seja T a interseção da reta
que passa por Q e E com o lado AB. 

Sejam BP=z, quadrado de lado AB=L, TB=k, CQ=x e QR=y. Por semelhança de
triângulos verifique que : 

x/k =L/z e y/L =x/z donde x^2=ky. Agora por semelhança veja que 

y/AT= x/k ou seja ky=x.AT e como ky=x^2 temos que x=AT ou seja CQ=AT. 

Como CQ é paralelo a AT e congruentes, temos que o quadrilátero ACQT é
um paralelogramo e já que as diagonais do quadrado são perpendiculares
temos que QT é perpendicular a BD. 

Temos então que no triângulo BDQ, BC e QH( H é a interseção de QT com
BD); 

ou seja E é o ortocentro de BDQ; donde PD é perpendicular a BQ. 

Verifiquem se há algum erro, ok? 

Abraços 

Carlos Victor 

Em 23/11/2018 22:38, Vanderlei Nemitz escreveu: 

> Estamos aguardando o Carlos Victor... 
> :) 
> 
> Em sex, 23 de nov de 2018 18:14, Mauricio de Araujo 
>  Alguem conseguiu finalizar a demonstração? 
> 
> Em qua, 21 de nov de 2018 11:52, Vanderlei Nemitz  escreveu: 
> Hummm... 
> Parece que prolongando BF e DC, que se encontram num ponto Q, E é o 
> ortocentro do triângulo BDQ. 
> O desenho sugere isso. 
> Mas como mostrar isso? 
> 
> Em ter, 20 de nov de 2018 23:38, Carlos Victor  escreveu: 
> 
> Oi Vanderlei, 
> 
> Uma dica : tente mostrar que o ponto E é o ortocentro de um triângulo " 
> estratégico". É muito legal que você descubra sozinho 
> 
> Abraços 
> 
> Carlos Victor 
> 
> Em 20/11/2018 17:33, Vanderlei Nemitz escreveu: 
> Pessoal, o seguinte problema sai "tranquilamente" usando Geometria Analítica. 
> Tentei usar Geometria Plana, mas apenas girei bastante, sem concluir. Será 
> que é possível? 
> 
> Dado um ponto P situado no prolongamento do lado AB de um quadrado ABCD, 
> traçam-se as retas PC e PD. Pelo ponto E, intersecção de BC e PD, conduzimos 
> a reta AE cuja intersecção com PC é o ponto F. Provar que BF e PD são 
> perpendiculares. 
> -- 
> Esta mensagem foi verificada pelo sistema de antivrus e 
> acredita-se estar livre de perigo.

-- 
Esta mensagem foi verificada pelo sistema de antivírus e 
acredita-se estar livre de perigo. 
-- 
Esta mensagem foi verificada pelo sistema de antivírus e 
acredita-se estar livre de perigo. 
-- 
Esta mensagem foi verificada pelo sistema de antivrus e 
acredita-se estar livre de perigo. 
 
-- 
Esta mensagem foi verificada pelo sistema de antiv�rus e
 acredita-se estar livre de perigo.



[obm-l] Re: [obm-l] Re: [obm-l] Re: [obm-l] Demonstração com Geometria Plana?

2018-11-23 Por tôpico Carlos Victor
 

Desculpem, estou em trânsito. Até amanhã eu posto, ok ? 

Abraços 

Em 23/11/2018 18:05, Mauricio de Araujo escreveu: 

> Alguem conseguiu finalizar a demonstração? 
> 
> Em qua, 21 de nov de 2018 11:52, Vanderlei Nemitz  escreveu: 
> Hummm... 
> Parece que prolongando BF e DC, que se encontram num ponto Q, E é o 
> ortocentro do triângulo BDQ. 
> O desenho sugere isso. 
> Mas como mostrar isso? 
> 
> Em ter, 20 de nov de 2018 23:38, Carlos Victor  escreveu: 
> 
> Oi Vanderlei, 
> 
> Uma dica : tente mostrar que o ponto E é o ortocentro de um triângulo " 
> estratégico". É muito legal que você descubra sozinho 
> 
> Abraços 
> 
> Carlos Victor 
> 
> Em 20/11/2018 17:33, Vanderlei Nemitz escreveu: 
> Pessoal, o seguinte problema sai "tranquilamente" usando Geometria Analítica. 
> Tentei usar Geometria Plana, mas apenas girei bastante, sem concluir. Será 
> que é possível? 
> 
> Dado um ponto P situado no prolongamento do lado AB de um quadrado ABCD, 
> traçam-se as retas PC e PD. Pelo ponto E, intersecção de BC e PD, conduzimos 
> a reta AE cuja intersecção com PC é o ponto F. Provar que BF e PD são 
> perpendiculares. 
> -- 
> Esta mensagem foi verificada pelo sistema de antivrus e 
> acredita-se estar livre de perigo.

-- 
Esta mensagem foi verificada pelo sistema de antivírus e 
acredita-se estar livre de perigo. 
-- 
Esta mensagem foi verificada pelo sistema de antivrus e 
acredita-se estar livre de perigo. 
 
-- 
Esta mensagem foi verificada pelo sistema de antiv�rus e
 acredita-se estar livre de perigo.



[obm-l] Re: [obm-l] Demonstração com Geometria Plana?

2018-11-20 Por tôpico Carlos Victor
 

Oi Vanderlei, 

Uma dica : tente mostrar que o ponto E é o ortocentro de um triângulo "
estratégico". É muito legal que você descubra sozinho 

Abraços 

Carlos Victor 

Em 20/11/2018 17:33, Vanderlei Nemitz escreveu: 

> Pessoal, o seguinte problema sai "tranquilamente" usando Geometria Analítica. 
> Tentei usar Geometria Plana, mas apenas girei bastante, sem concluir. Será 
> que é possível? 
> 
> Dado um ponto P situado no prolongamento do lado AB de um quadrado ABCD, 
> traçam-se as retas PC e PD. Pelo ponto E, intersecção de BC e PD, conduzimos 
> a reta AE cuja intersecção com PC é o ponto F. Provar que BF e PD são 
> perpendiculares. 
> -- 
> Esta mensagem foi verificada pelo sistema de antivrus e 
> acredita-se estar livre de perigo.

 
-- 
Esta mensagem foi verificada pelo sistema de antiv�rus e
 acredita-se estar livre de perigo.



[obm-l] Re: [obm-l] Valor mínimo

2018-09-08 Por tôpico Carlos Victor
 

Olá Daniel, 

Esta questão saiu da original que foi de uma Olimpíada de Leningrad em
1988 cujo enunciado era : 

a,b,c e d reais positivos; prove que 1/a+1/b+4/c+16/d >= 64/(a+b+c+d). 

Tome (1/a+1/b+4/c+16/d).(a+b+c+d)= 22+(a/b+b/a)+2(2a/c+
c/2a)+4(4a/d+d/4a)+2(2b/c+c/2b)+4(4b/d+d/4b)+8(2c/d+d/2c)>=22+2+2.2+4.2+2.2+4.2+8.2=64


Abraços 

Carlos Victor 

Em 08/09/2018 9:31, Daniel Quevedo escreveu: 

> Se A, B, C e D são reais positivos então o valor mínimo de 1/A + 1/B + 4/C + 
> 16/D é igual a: 
> A) 1/(A + B +C+D) 
> B) 16/(A + B +C+D) 
> C) 2/(A + B +C+D) 
> D) 64/(A + B +C+D) 
> E) 4/(A + B +C+D) 
> 
> R: d -- 
> 
> Fiscal: Daniel Quevedo 
> -- 
> Esta mensagem foi verificada pelo sistema de antivrus e 
> acredita-se estar livre de perigo.

 
-- 
Esta mensagem foi verificada pelo sistema de antiv�rus e
 acredita-se estar livre de perigo.



[obm-l] Re: [obm-l] Equação 4 grau

2018-06-26 Por tôpico Carlos Victor
 

Oi daniel, 

Faça (x^2+1)^2 =2(x+1)^2 e . 

Abraçõs 

Carlos Victor 

Em 26/06/2018 15:09, Daniel Quevedo escreveu: 

> As raizes reais da equação x^4 -4x=1 pertencem ao intervalo: 
> A) (1,11) 
> B) (2, 12) 
> C) (3, 13) 
> D) (4, 14) 
> E) ( 5, 15) 
> 
> R: c -- 
> 
> Fiscal: Daniel Quevedo 
> -- 
> Esta mensagem foi verificada pelo sistema de antivrus e 
> acredita-se estar livre de perigo.

 
-- 
Esta mensagem foi verificada pelo sistema de antiv�rus e
 acredita-se estar livre de perigo.



[obm-l] Re: [obm-l] Re: [obm-l] Re: [obm-l] Re: [obm-l] Equação Funcional

2018-06-12 Por tôpico Carlos Victor
 

Olá pessoal, 

Devemos ser cuidadosos com este livro. Há muitas respostas
inconsistentes no gabarito. 

Carlos Victor 

Em 12/06/2018 14:00, Pedro José escreveu: 

> Boa tarde!
> 
> Acho estranho, pois fui compondo g(x) com g(x), sendo g(x)=(1-x)/x e, 
> verifiquei que nunca vai dar a identidade. Dá o quociente de duas funções 
> afins e portanto nunca dará x. Por curiosidade, os coeficientes dos 
> polinômios de primeiro grau são, em módulo, termos da sequência de Fibonacci. 
> E continuo achando estranho pois se supormos que f(x)=x^3-x^2-1/[x(x-1)] é a 
> solução temos:
> 
> f(x)+f((1-x)/x)=f(x)+f(1/x-1)= x^3-x^2-1/[x(x-1)] + 1/x^3 - 4/x^2 + 5/x - 2 + 
> x^2/(2x^2-3x+1)
> 
> só que pela definição tem que ser igual a 1+x, não vejo como será cancelado 
> esse termo em x^3, por exemplo. Será que fiz barbeiragem nesse 
> desenvolvimento?
> 
> Pois, me parece que algo de errado, ou com o enunciado ou com a solução.
> 
> Não satisfeito, peguei x = (-1+ raiz(5))/2, que é uma das raízes de (1-x)/x=x
> 
> Pela definição temos 2f((-1+raiz(5))/2) = 1 + (-1+raiz(5)/2) ==> 
> f((-1+raiz(5))/2) = (1+raiz(5))/4
> 
> mas aplicando a solução proposta:
> 
> f((-1+raiz(5)/2)) = 1/8 (-1 + 3raiz(5) -15 +raiz(5)^3) - 1/4(1 -2raiz(5) +5) 
> - 1 = 1/8 ( -12+raiz(5)+raiz(5)^3 <> (1+raiz(5))/4, que já poderia ser visto, 
> de cara, pela existência do termo raiz(5)^3.
> 
> O problema não está fechando, creio eu. 
> Ou defeito na proposição ou no resultado. 
> Saudações, PJMS
> 
> Em 11 de junho de 2018 23:31, Jeferson Almir  
> escreveu:
> 
> Esse é o problema 2901 do livro Problemas Selecionados de Matemática ( Gandhi 
> ) 
> E resposta que ele diz é 
> R: x^3 - x^2 - 1 / x(x-1) 
> 
> Em seg, 11 de jun de 2018 às 12:15, Jeferson Almir  
> escreveu: 
> 
> Isso mesmo Ralph eu sei fazer g(x) = (x-1)/x 
> 
> Em seg, 11 de jun de 2018 às 11:33, Ralph Teixeira  
> escreveu: 
> Puxa, se fosse g(x)=(x-1)/x ali dentro do segundo termo, eu sabia fazer 
> rápido... :( Era só escrever y=g(x), z=g(y), e então: 
> f(x)+f(y)=1+x 
> f(y)+f(z)=1+y 
> f(z)+f(x)=1+z 
> pois é fácil ver que g(z)=g(g(g(x)))=x. Resolvendo esse sisteminha, 
> acharíamos f(x). 
> 
> Porém, com esse enunciado... Hm, alguém confere aqui o raciocínio abaixo, 
> porque acho que eu consigo mostrar que **não dá** para resolver isso, mas 
> estou morrendo de sono, então provavelmente escrevi alguma bobagem imensa. 
> 
> Observe que g(x)=(1-x)/x é injetiva (e sua inversa é g^(-1)(y)=1/(1+y)). Dado 
> um x_0=a, crie a sequência {x_k} com k inteiro onde x_(k+1)=g(x_k) -- observe 
> que crio isto incluindo k negativo, o que é possível desde que nenhum dos 
> números da órbita seja 0 ou -1. Vou chamar o **conjunto** de valores {x_k} de 
> "órbita" do número a. 
> 
> Pois bem, a equação funcional só dá informações sobre os valores de f dentro 
> de cada órbita! Ela diz que f(x_k)+f(x_(k+1))=1+x_k (*), e mais nada, ou 
> seja, ela não relaciona os valores de f em órbitas distintas! Se a órbita é 
> infinita, isto é, se os x_k são todos distintos, você pode ESCOLHER f(a) como 
> quiser e calcular os outros f(x_k) usando (*) como recorrência. 
> 
> Agora você me pergunta: porque a órbita não fecha? Bom, você tem razão, para 
> vários valores de "a" a órbita fecha, isto é, poderia ser x_P=x_0=a para 
> algum P<>0... Mas a equação x_P=a quer dizer g(g(g(...g(a))...)=a, que é uma 
> equação quadrática (né?), e portanto tem no máximo 2 raízes reais. Então, 
> mesmo que consideremos todos os P possíveis, o conjunto dos a que fazem a 
> órbita fechar é enumerável... Bom, os reais não são enumeráveis, então há 
> várias órbitas infinitas Acho. 
> 
> Abraço, Ralph. 
> 
> P.S.: Se eu tivesse bom senso, conferia isso antes de mandar para a lista... 
> Ah, dane-se, mesmo que eu esteja errado este tipo de raciocínio é 
> interessante, não? 
> P.S.2: Se o enunciado falar que f é *contínua*, aí talvez dê para fazer algo 
> usando o limite de x_k... 
> 
> On Mon, Jun 11, 2018 at 8:32 AM Jeferson Almir  
> wrote: 
> 
> Seja f(x) uma função real definida em R -{0,1} tal que 
> 
> f(x) + f( 1-x | x ) =1 + x determine f (x) . 
> 
> Obs: ( 1-x | x) é 1-x dividido por x . 
> -- 
> Esta mensagem foi verificada pelo sistema de antivírus e 
> acredita-se estar livre de perigo. 
> -- 
> Esta mensagem foi verificada pelo sistema de antivírus e 
> acredita-se estar livre de perigo.

-- 
Esta mensagem foi verificada pelo sistema de antivírus e 
acredita-se estar livre de perigo. 
-- 
Esta mensagem foi verificada pelo sistema de antivrus e 
acredita-se estar livre de perigo. 
 
-- 
Esta mensagem foi verificada pelo sistema de antiv�rus e
 acredita-se estar livre de perigo.



Re: [obm-l] Limite

2018-03-19 Por tôpico Carlos Victor
 

Oi Vanderlei, 

Use a equivalência de Stirling : 

n! ~ n^n.e^(-n).sqrt(2pi.n) e que lim(n^(1/n)=1 e o resultado será 1/e. 

Abraços 

Carlos Victor 

Em 19/03/2018 12:27, Vanderlei Nemitz escreveu: 

> Bom dia! 
> Eu resolvi o limite a seguir de um modo muito complicado e encontrei 1/e. 
> 
> Alguém conhece alguma solução? 
> 
> lim [n!/n^n]^(1/n), quando n tende ao infinito. 
> 
> Muito obrigado! 
> -- 
> Esta mensagem foi verificada pelo sistema de antivrus e 
> acredita-se estar livre de perigo.

 
-- 
Esta mensagem foi verificada pelo sistema de antiv�rus e
 acredita-se estar livre de perigo.



Re: [obm-l] Algebra (Polinomios)

2017-07-10 Por tôpico Carlos Victor
 

Oi Douglas, faça o seguinte: 

p(x) = (x^2+x+1)^40 = [x(x+1)+1]^40 e tomando y = x(x+1) e desenvolva o
binômo de Newton 

(y+1)^40 = [y+1)^39](y+1). Observe que os três últimos do
desenvolvimento dentro dos colchetes serão : 741y^2+39y+1, pois os
anteriores serão divisíveis por (x+1)^3. 

Basta então encontrar o resto de (741y^2+39y+1)(y+1) por (x+1)^3. 

Seja g(y) = (741y^2+39y+1)(y+1) com y = x(x+1). Como estamos dividindo
por x^3+3x^2+3x+1, basta substituirmos x^3 por -3x^2-3x-1 no
desenvolvimento de g(y). 

Fazendo algumas continhas (confira), encontramos o resto igual a
820x^2+1600x+781. 

Abraços 

Carlos Victor 

Em 10/07/2017 20:37, Douglas Oliveira de Lima escreveu: 

> Encontrar o resto da divisão do polinomio (x^2+x+1)^40 por (x+1)^3. 
> 
> Obs: Sem usar derivadas. 
> 
> Douglas Oliveira. 
> -- 
> Esta mensagem foi verificada pelo sistema de antivrus e 
> acredita-se estar livre de perigo.

 
-- 
Esta mensagem foi verificada pelo sistema de antiv�rus e
 acredita-se estar livre de perigo.



[obm-l] Re: [obm-l] Polinômios

2017-05-27 Por tôpico Carlos Victor
 

Oi Wanderlei, 

seja o resto dado por R(x)=ax^3+bx^2+cx+d. 

Onde tiver x^2 em R(x) substitua por (-x-1) e force ser igual a -x+1;
encontrando : 

c-b=-1 e a+d-b=1. Depois onde tiver x^2 substitua por(x-1) e force ser
igual a 3x+5; encontrando b+c=3 e d-b-a=5. 

conclusão : a=-2, b=2 , c=1 e d=5. 

Abraços 

Carlos Victor 

Em 27/05/2017 11:17, Vanderlei Nemitz escreveu: 

> Bom dia! 
> 
> Alguém poderia dar uma ideia na seguinte questão? Já tentes algumas 
> estratégias, mas sem êxito. 
> 
> UM POLINÔMIO P(X) DIVIDIDO POR X^2 + X + 1 DÁ RESTO -X + 1 E DIVIDIDO POR X^2 
> -X + 1 DÁ RESTO 3X + 5. QUAL O RESTO DA DIVISÃO DE P(X) POR X^4 + X^2 + 1? 
> 
> A resposta que tenho é -2X^3 + 2X^2 + X + 5. 
> 
> Obrigado! 
> 
> Vanderlei 
> -- 
> Esta mensagem foi verificada pelo sistema de antivrus e 
> acredita-se estar livre de perigo.

 
-- 
Esta mensagem foi verificada pelo sistema de antiv�rus e
 acredita-se estar livre de perigo.



[obm-l] Re: [obm-l] [obm-l] Questão Geometria

2016-10-10 Por tôpico Carlos Victor
 

Olá Vinicius, 

Seja R a intersecção de AO com BC. Seja T a intersecção da bissetriz de
 Será que alguém poria me ajudar na seguinte questão? 
> 
> * 
> 
> (Belarus) Seja O o centro do círculo ex-inscrito do triângulo ABC oposto ao 
> vértice A. Seja M o ponto médio de AC e seja P a intersec ̧ão das retas MO e 
> BC. Prove que se ∠BAC = 2∠ACB, então AB = BP. 
> 
> -- 
> Esta mensagem foi verificada pelo sistema de antivrus e 
> acredita-se estar livre de perigo.
 
-- 
Esta mensagem foi verificada pelo sistema de antiv�rus e
 acredita-se estar livre de perigo.



Re: [obm-l] Re: Geometria

2016-09-11 Por tôpico Carlos Victor
 

Oi Jeferson, 

Tome E sobre BD tal que o ângulo EAB seja 30º. Observe que o ângulo ADB
é igual a 100º e que o ângulo DAE é igual a 20º. Daí o ângulo AED é
igual a 60º. Como E está na bissetriz de ACB, então o ângulo AEC é igual
a 120º. Observe agora que D é o ponto de encontro das bissetrizes
internas do triângulo AEC e consequentemente o ângulo BDC é igual a
110º. 

Abraços 

Carlos Victor 

Em 10/09/2016 17:34, Jeferson Almir escreveu: 

> Olá pessoa queria uma ajuda nessa questão
> 
> A figura em anexo mostra um triângulo _ABC_. _D_ é um ponto interior onde a 
> medida dos ângulos _CAD_, _ABD_, _CBD_, e _BAD_ são 20º, 30º, 40º e 50º , 
> respectivamente. Encontre a medida do ângulo _BDC_. 
> 
> Em 28 de agosto de 2016 18:31, Jeferson Almir <jefersonram...@gmail.com> 
> escreveu:
> 
>> Olá pessoa queria uma ajuda nessa questão
>> 
>> A figura em anexo mostra um triângulo _ABC_. _D_ é um ponto interior onde a 
>> medida dos ângulos _CAD_, _ABD_, _CBD_, e _BAD_ são 20º, 30º, 40º e 50º , 
>> respectivamente. Encontre a medida do ângulo _BDC_.
> 
> -- 
> Esta mensagem foi verificada pelo sistema de antivrus e 
> acredita-se estar livre de perigo.
 
-- 
Esta mensagem foi verificada pelo sistema de antiv�rus e
 acredita-se estar livre de perigo.



[obm-l] Re: [obm-l] Re: [obm-l] Dúvida sobre a Obm U

2016-07-25 Por tôpico Carlos Victor
 

Oi Otávio, 

Você já viu a Revista Matemática Universitária da SBM ? 

Em 25/07/2016 10:09, Otávio Araújo escreveu: 

> Pois é, se algum professor com experiência em olimpíadas, como o Nicolau por 
> exemplo, respondesse minha pergunta seria de grande ajuda 
> 
> Em 24 de jul de 2016, às 23:25, Israel Meireles Chrisostomo 
>  escreveu:
> 
> Boa pergunta, eu tambÃ(c)m tenho interesse em participar da OBM U e gostaria 
> de umas dicas 
> 
> Em 16 de julho de 2016 13:29, Otávio Araújo  
> escreveu:
> Galera, gostaria que vocês me dessem dicas de o que estudar, como estudar 
> e por quais livros e materiais estudar para a prova da Obm nível 
> universitário...
> Estou muito interessado em participar, mas fico meio confuso por onde 
> estudar...
> Por favor me ajudem
> --
> Esta mensagem foi verificada pelo sistema de antivírus e
> Â acredita-se estar livre de perigo.
> 
> =
> Instruções para entrar na lista, sair da lista e usar a lista em
> http://www.mat.puc-rio.br/~obmlistas/obm-l.html [1]
> =
> 
> -- 
> Esta mensagem foi verificada pelo sistema de antivírus e 
> acredita-se estar livre de perigo.

-- 
Esta mensagem foi verificada pelo sistema de antivrus e 
acredita-se estar livre de perigo. 

Links:
--
[1] http://www.mat.puc-rio.br/~obmlistas/obm-l.html

-- 
Esta mensagem foi verificada pelo sistema de antiv�rus e
 acredita-se estar livre de perigo.



[obm-l] Re: [obm-l] [OFF] Aneis Adélicos (Adèles)

2016-05-24 Por tôpico Carlos Victor
 

OI Listeiro.
Dê uma olhada neste
material:http://www.pg.im.ufrj.br/teses/Matematica/Mestrado/319.pdf [1] 

Abraços 

Carlos Victor 

Em 23/05/2016 12:04, Listeiro 037 escreveu: 

> Saudações a todos.
> 
> Esbarrei com um conceito algébrico chamado Adele. Não encontrei
> material claro sobre este conceito. Alguém conhece algum? Desde já
> agradeço.
 

Links:
--
[1] http://www.pg.im.ufrj.br/teses/Matematica/Mestrado/319.pdf

-- 
Esta mensagem foi verificada pelo sistema de antiv�rus e
 acredita-se estar livre de perigo.



Re: [obm-l] Combinatoria

2016-02-01 Por tôpico Carlos Victor
 

Olá, 

A figura(cartela) é um retângulo dividido em seis quadrados, tendo dois
quadrados por coluna. 

Pacini 

Em 01/02/2016 3:06, Israel Meireles Chrisostomo escreveu:

Em 31 de janeiro de 2016 22:43, Pacini Bores 
escreveu:

> Olá pessoal, 
> 
> Creio que a figura não apareceu. É um retângulo dividido em seis quadrados, 
> tendo dois quadrados por coluna. 
> 
> Obrigado 
> 
> Pacini 
> 
> Em 31/01/2016 14:30, Pacini Bores escreveu: 
> 
>> Olá pessoal , poderia me ajudar na questão abaixo ? 
>> 
>> Cada cartela de uma coleção é formado por seis quadrados colorodos, 
>> justapostos como indica a figura abaixo: 
>> 
>> Em cada cartela, dois quadrados foram coloridos de azul, dois de verde e 
>> dois de rosa. A coleção apresenta as possibilidades de distribuição dessas 
>> cores nas cartelas nas condições citadas e não existem cartelas com a mesma 
>> distribuição de cores. Retirando-se ao acaso uma cartela, determine a 
>> probabilidade de que somente uma coluna apresente os quadrados de mesma cor. 
>> 
>> Agradeço desde já qualquer comentário 
>> 
>> Pacini
 

[obm-l] Re: [obm-l] Máximo absoluto de f(x) =( 5x -1) / (x^2 + 1)

2015-12-14 Por tôpico Carlos Victor
 

Oi Pedro, observe inicialmente que o campo de definição é o conjunto dos
reais. 

Chame y = (5x-1)/(x^2+1) e monte uma equação do segundo grau em x. Faça
o delta maior do que ou igual a zero. 

Abraços 

Carlos Victor 

Em 13/12/2015 22:07, Pedro Chaves escreveu: 

> Caros Colegas,
> 
> Como provar, sem recorrer a limites nem a derivadas, que existe o máximo 
> absoluto da função f(x) = (5x - 1) / (x^2 + 1), definida para todo x real?
> 
> Abraços do Pedro Chaves
> --- 
> -- 
> Esta mensagem foi verificada pelo sistema de antivírus e 
> acredita-se estar livre de perigo.
 
-- 
Esta mensagem foi verificada pelo sistema de antiv�rus e
 acredita-se estar livre de perigo.



[obm-l] Re: [obm-l] Soma de números compostos

2015-12-13 Por tôpico Carlos Victor
 

Oi Marcone, 

Para n maior do que ou igual a 1, temos: 

i)11+3n = 8+3(n+1) 

ii)11+3n+1 = 9+3(n+1) 

iii) 11+3n+2 = 10+3(n+1) 

Faltando : 12 =8+4 e 13 = 9+4. 

Abraços 

Carlos Victor 

Em 11/12/2015 23:36, marcone augusto araújo borges escreveu: 

> Mostre que todo inteiro n > 11 pode ser escrito como soma de números 
> compostos positivos 
> 
> para n par : n = 11 + 2t-1 = 4 + [2(t + 3)] 
> mas... 
> -- 
> Esta mensagem foi verificada pelo sistema de antivírus e 
> acredita-se estar livre de perigo.
 
-- 
Esta mensagem foi verificada pelo sistema de antiv�rus e
 acredita-se estar livre de perigo.



Re: [obm-l] Primo?

2015-11-25 Por tôpico Carlos Victor
 

Não. 

Observe um dos emails do Pacini. 

(2^83-1)(2^83+1)=2^166-1; por Fermat...; daí ele tentou verificar se 167
é fator do número pedido. 

Abraços 

Carlos victor 

Em 24/11/2015 20:13, Mauricio de Araujo escreveu: 

> Só para ser chato, o primo 167 caiu do céu? rsss (sem ofensas) 
> 
> No enunciado original não é mencionado o primo 167... 
> 
> Em 24 de novembro de 2015 16:48, Matheus Secco <matheusse...@gmail.com> 
> escreveu:
> 
> Acredito que você possa usar resíduos quadráticos: 
> 
> (2 legendre p) = (-1)^(p^2-1)/8 
> 
> (2 legendre p) == 2^(p-1)/2 (mód p) 
> 
> Para p = 167, temos que (167^2-1)/8 é par. Logo (2 legendre 167) = 1. 
> Com isso, obtemos que 2^83 == 1 (mód 167). 
> 
> Abraços 
> 
> 2015-11-24 10:16 GMT-02:00 Pacini Bores <pacini.bo...@globo.com>:
> 
> Olá Marcone, 
> 
> Observe que 2^166-1 é divisível por 167; logo um dos fatores de 
> (2^83-1)(2^83+1) divide 167, já que 167 é primo. Só estou tentando provar que 
> é 2^83-1, que ainda não consegui. 
> 
> Pacini 
> 
> Em 24/11/2015 7:32, marcone augusto araújo borges escreveu: 
> Mostre que 2^83 - 1 não é primo 
> -- 
> Esta mensagem foi verificada pelo sistema de antivírus e 
> acredita-se estar livre de perigo. 
> -- 
> Esta mensagem foi verificada pelo sistema de antivírus e 
> acredita-se estar livre de perigo.

-- 
Esta mensagem foi verificada pelo sistema de antivírus e 
acredita-se estar livre de perigo. 

 -- 

Abraços 

oɾnɐɹɐ ǝp oıɔıɹnɐɯ 

-- 
Esta mensagem foi verificada pelo sistema de antivrus e 
acredita-se estar livre de perigo. 
-- 
Esta mensagem foi verificada pelo sistema de antiv�rus e
 acredita-se estar livre de perigo.



Re: [obm-l] Conicas

2015-10-31 Por tôpico Carlos Victor
 

Usando a ideia do Pacini, observe que dy/dx =0 para x=1 e y=0; dy/dx não
existe para x =-1/2 e que o eixo de simetria passa pelo ponto(-1/2,0) ;
ou seja o eixo de simetria é dado por y= -x-1/2. Fazendo a intersecção
dessa reta com a curva dada, encontramos x=-1/8 e y= -3/8, que são as
coordenadas do vértice. 

Abraços 

Carlos Victor 

Em 29/10/2015 23:01, Douglas Oliveira de Lima escreveu: 

> Olá caros amigos, gostaria de uma ajuda no seguinte problema: 
> 
> PROBLEMA: Encontrar a abscissa da parábola de equação 
> x^2+2xy+y^2-2x+4y+1=0. 
> 
> OBS: Essa questão caiu na prova do ITA acho que de 2012, e vi uma solução que 
> envolvia limites do qual não compreendi muito bem. 
> Sei portanto como usar a rotação de eixos e também através de diagonalização. 
> Mas gostaria de saber se existe outro modo de chegar a tal abscissa. 
> 
> Desde já obrigado. 
> Forte abraço do Douglas Oliveira. 
> -- 
> Esta mensagem foi verificada pelo sistema de antivrus e 
> acredita-se estar livre de perigo.
 
-- 
Esta mensagem foi verificada pelo sistema de antiv�rus e
 acredita-se estar livre de perigo.



Re: [obm-l] Limites

2015-09-08 Por tôpico Carlos Victor
Oi  Israel,
lim(n!/((n^n).(e^(-n)).(sqrt(2.pi.n))) = 1( relação de Stirling)  e use  o
fato de que lim (n^(1/n))=1.

Abraços

Carlos  Victor

Em 8 de setembro de 2015 21:03, Israel Meireles Chrisostomo <
israelmchrisost...@gmail.com> escreveu:

> Como posso provar de forma simples que ((2n)!/(n!)²)^{1/n}=4?Estou
> dependendo desse resultado para calcular um outro limite...
>
>
> --
> Esta mensagem foi verificada pelo sistema de antivírus e
> acredita-se estar livre de perigo.

-- 
Esta mensagem foi verificada pelo sistema de antiv�rus e
 acredita-se estar livre de perigo.



[obm-l] Re: [obm-l] indução

2015-06-28 Por tôpico Carlos Victor
Oi Marcone, irei resumir .

Inicialmente a prova de que  n^33^n ou igual. Por indução:

3^(n+1) = 3.3^n  ou igual  que 3.n^3 = n^3+3n^2+3n + (n-3).n^2 + (n^2-3).n
 n^3+3n^2+3n+1 = (n+1)^3.

Suponha agora  que mn , então  m^(1/n) n^(1/n)  ou igual a 3^(1/3), ok ?

PS:
Esta questão foi da AMM, 1970,p 768, problem E2190, proposed by Harry
Pollard, Purdue University , solved by Charles Wexler, Arizona State
University, and 118 others.

Abraços

Carlos  Victor

Em 28 de junho de 2015 11:31, rigillesbmene...@gmail.com escreveu:

 Qual a necessidade de escrever n^1 ao invés de n? É algo da questão
 mesmo?

 Enviado do meu iPhone

 Em 28/06/2015, às 11:17, marcone augusto araújo borges 
 marconeborge...@hotmail.com escreveu:

 Prove por indução que n^1/n  = 3^1/3, para n  = 2. Mostre que um dos
 números
 n^1/m ou m^1/n é maior que ou igual a 3, m e  naturais

 --
 Esta mensagem foi verificada pelo sistema de antivírus e
 acredita-se estar livre de perigo.


 --
 Esta mensagem foi verificada pelo sistema de antivírus e
 acredita-se estar livre de perigo.


-- 
Esta mensagem foi verificada pelo sistema de antiv�rus e
 acredita-se estar livre de perigo.



[obm-l] Re: [obm-l] indução

2015-06-28 Por tôpico Carlos Victor
Observar que o enunciado é 3^(1/3), ok ?

Em 28 de junho de 2015 12:03, Carlos Victor victorcar...@globo.com
escreveu:

 Oi Marcone, irei resumir .

 Inicialmente a prova de que  n^33^n ou igual. Por indução:

 3^(n+1) = 3.3^n  ou igual  que 3.n^3 = n^3+3n^2+3n + (n-3).n^2 +
 (n^2-3).n  n^3+3n^2+3n+1 = (n+1)^3.

 Suponha agora  que mn , então  m^(1/n) n^(1/n)  ou igual a 3^(1/3), ok ?

 PS:
 Esta questão foi da AMM, 1970,p 768, problem E2190, proposed by Harry
 Pollard, Purdue University , solved by Charles Wexler, Arizona State
 University, and 118 others.

 Abraços

 Carlos  Victor

 Em 28 de junho de 2015 11:31, rigillesbmene...@gmail.com escreveu:

 Qual a necessidade de escrever n^1 ao invés de n? É algo da questão
 mesmo?

 Enviado do meu iPhone

 Em 28/06/2015, às 11:17, marcone augusto araújo borges 
 marconeborge...@hotmail.com escreveu:

 Prove por indução que n^1/n  = 3^1/3, para n  = 2. Mostre que um dos
 números
 n^1/m ou m^1/n é maior que ou igual a 3, m e  naturais

 --
 Esta mensagem foi verificada pelo sistema de antivírus e
 acredita-se estar livre de perigo.


 --
 Esta mensagem foi verificada pelo sistema de antivírus e
 acredita-se estar livre de perigo.




-- 
Esta mensagem foi verificada pelo sistema de antiv�rus e
 acredita-se estar livre de perigo.



[obm-l] Re: [obm-l] Área da Cicloide

2015-05-24 Por tôpico Carlos Victor
Oi  Eduardo, existe um texto no endereço a seguir. Verifique se é o que
você deseja.
http://www.apm.pt/apm/foco98/activ9.html

Abraços

Carlos  Victor

Em 24 de maio de 2015 18:46, Eduardo Henrique dr.dhe...@outlook.com
escreveu:

 Eu lendo um livro de história da matemática vi que Torricelli e Wren
 conseguiram demonstrar que a área sob um arco de cicloide é 3x a área do
 circulo que a gera utilizando o método da exaustão! Alguém saberia me
 indicar onde conseguir essas demonstrações ou até mesmo me dar uma luz em
 como faze-la?

 Att

 Eduardo

 --
 Esta mensagem foi verificada pelo sistema de antivírus e
 acredita-se estar livre de perigo.


-- 
Esta mensagem foi verificada pelo sistema de antiv�rus e
 acredita-se estar livre de perigo.



Re: [obm-l] Re:

2015-03-30 Por tôpico Carlos Victor
Olá Gabriel, esta é do livro do Gandhi :

(x^2+2)^2 = 4(x-2)^2 e daí .

Abraços

Carlos  Victor

Em 30 de março de 2015 07:16, Carlos Victor victorcar...@globo.com
escreveu:

 Tente completar quadrados.

 Abraços

 Carlos Victor

 Em 29 de março de 2015 21:27, Gabriel Tostes gtos...@icloud.com
 escreveu:

 AlguÃĐm me ajuda a responder?
 determine as raízes reais da equaçÃĢo:
 X^4 + 16x - 12 = 0
 --
 Esta mensagem foi verificada pelo sistema de antivírus e
  acredita-se estar livre de perigo.


 =
 Instruįões para entrar na lista, sair da lista e usar a lista em
 http://www.mat.puc-rio.br/~obmlistas/obm-l.html
 =



 --
 Esta mensagem foi verificada pelo sistema de antivírus e
 acredita-se estar livre de perigo.

-- 
Esta mensagem foi verificada pelo sistema de antiv�rus e
 acredita-se estar livre de perigo.



Re: [obm-l] Tabuleiro 3x3 com 4 cores

2015-03-30 Por tôpico Carlos Victor
Acredito que  ideia do Bob Roy é o mais rápida para obter a solução.

Carlos  Victor

Em 30 de março de 2015 10:39, Pacini Bores pacini.bo...@globo.com
escreveu:

 Sim Pedro, esta é uma solução; ou seja, há possibilidade de se usar até
 quatro cores.

 Pacini

 Em 30 de março de 2015 10:23, Pedro José petroc...@gmail.com escreveu:

 Bom dia!

 Uma dúvida há necessidade de se usar as quatro cores ou há a
 possibilidade de se usar até quatro cores?

 Por exemplo,

 0 1 0
 1 0 1
 0 1 0

 onde 0 e 1 representam duas cores distintas, seria uma solução?

 Saudações,

 PJMS





 Em 29 de março de 2015 11:26, Bob Roy bob...@globo.com escreveu:

 Olá, O melhor para este problema é utlizar  o que o grande mestre
 Morgado falava : devemos inicialmente eliminar as dificuldades.

 Considerando uma matriz 3x3 , temos que os quadradinhos a12, a21, a23 e
 a32 não poderão ter todas as cores diferentes.

 Comece fazendo a análise com  duas cores iguais, três cores iguais e
 depois quatro cores iguais para essas posições.

 A análise ficará menos trabalhosa .

 Farei as contas e depois eu posto o resultado.

 Roy


 Em 28 de março de 2015 10:22, Carlos Victor victorcar...@globo.com
 escreveu:

 Comece pelo centro e pelas laterais, isto deve diminuir as
 dificuldades. Abrirão vários casos para serem analisados.

 E se  não me engano, esta questão tem como origem  não considerando os
 quadrados pelos vértices com as mesmas cores. Neste  caso a análise fica
 mais silmplificada.

 Abraços

 Carlos Victor

 Em 28 de março de 2015 09:38, Pacini Bores pacini.bo...@globo.com
 escreveu:

 Olá pessoal,  como pensar nesta ?

 De quantas maneiras podemos pintar um tabuleiro 3x3 com 4 cores de tal
 forma que não tenhamos cores adjacentes ?

 Nota : em diagonal não é considerado adjacente.

 Agradeço desde já

 Pacini.

 --
 Esta mensagem foi verificada pelo sistema de antivírus e
 acredita-se estar livre de perigo.



 --
 Esta mensagem foi verificada pelo sistema de antivírus e
 acredita-se estar livre de perigo.



 --
 Esta mensagem foi verificada pelo sistema de antivírus e
 acredita-se estar livre de perigo.



 --
 Esta mensagem foi verificada pelo sistema de antivírus e
 acredita-se estar livre de perigo.



 --
 Esta mensagem foi verificada pelo sistema de antivírus e
 acredita-se estar livre de perigo.


-- 
Esta mensagem foi verificada pelo sistema de antiv�rus e
 acredita-se estar livre de perigo.



[obm-l] Re:

2015-03-30 Por tôpico Carlos Victor
Tente completar quadrados.

Abraços

Carlos Victor

Em 29 de março de 2015 21:27, Gabriel Tostes gtos...@icloud.com escreveu:

 AlguÃĐm me ajuda a responder?
 determine as raízes reais da equaçÃĢo:
 X^4 + 16x - 12 = 0
 --
 Esta mensagem foi verificada pelo sistema de antivírus e
  acredita-se estar livre de perigo.


 =
 Instruįões para entrar na lista, sair da lista e usar a lista em
 http://www.mat.puc-rio.br/~obmlistas/obm-l.html
 =


-- 
Esta mensagem foi verificada pelo sistema de antiv�rus e
 acredita-se estar livre de perigo.



Re: [obm-l] Tabuleiro 3x3 com 4 cores

2015-03-28 Por tôpico Carlos Victor
Comece pelo centro e pelas laterais, isto deve diminuir as dificuldades.
Abrirão vários casos para serem analisados.

E se  não me engano, esta questão tem como origem  não considerando os
quadrados pelos vértices com as mesmas cores. Neste  caso a análise fica
mais silmplificada.

Abraços

Carlos Victor

Em 28 de março de 2015 09:38, Pacini Bores pacini.bo...@globo.com
escreveu:

 Olá pessoal,  como pensar nesta ?

 De quantas maneiras podemos pintar um tabuleiro 3x3 com 4 cores de tal
 forma que não tenhamos cores adjacentes ?

 Nota : em diagonal não é considerado adjacente.

 Agradeço desde já

 Pacini.

 --
 Esta mensagem foi verificada pelo sistema de antivírus e
 acredita-se estar livre de perigo.

-- 
Esta mensagem foi verificada pelo sistema de antiv�rus e
 acredita-se estar livre de perigo.



Re: [obm-l] Problema IMO

2015-03-10 Por tôpico Carlos Victor
Oi Israel, no link

http://www.artofproblemsolving.com/wiki/index.php/1985_IMO_Problems/Problem_4,
 vc encontra a solução, ok ?


Abraços


Carlos  Victor



Em 10 de março de 2015 21:46, Israel Meireles Chrisostomo 
israelmchrisost...@gmail.com escreveu:

 Alguém poderia me ajudar nessa questão envolvendo o princípio da casa dos
 pombos?
 Dado um conjunto M com 1985 inteiros positivos distintos, nenhum dos
 quais tem divisores maiores do que 23, mostre que há 4 elementos em M cujo
 produto é uma quarta potência.
 Pensei em usar que de 2 a 23 tem 9 números primos, mas não sei bem como
 aplicar o princípio, se alguém pudesse me ajudar serei grato.

 --
 Esta mensagem foi verificada pelo sistema de antivírus e
 acredita-se estar livre de perigo.

-- 
Esta mensagem foi verificada pelo sistema de antiv�rus e
 acredita-se estar livre de perigo.



[obm-l] Re: [obm-l] Re: [obm-l] OBM NÍVEL 3 TERCEIRA FASE PRIMEIRO DIA

2014-11-17 Por tôpico Carlos Victor
Oi grande Douglas !!

Como sempre postando bons problemas para a nossa comunidade.

Vamos lá :

Sejam M,N,R e Q os incentros dos triângulos ABP, BPC, CPD e APD
respectivamente.

Sejam S, T, U e V  os incentros dos triângulos ABC,BCD, ACD e ABD
respectivamente.

1º) mostre que  MNRQ é um losango. Mostre também que os raios dos círculos
inscritos em ABC e ADC são iguais; da mesma forma dos triângulos ABD e BCD.

2º) depois mostre que  AM/MS =  AQ/QU  e que  SN/NC = RU/RT .

3º) como consequência SU é paralelo a BD e que VT é paralelo  a AC.

4º) mostre que mostre que MS/AM = UR/RC.

5º) mostre que o ângulo MSN = ângulo QUR ; da mesma forma  ângulo NTR  =
ângulo MVQ.

6º) conclua então que ASCU é um paralelogramo.

7º) conclua daí que pelo fato de PN = PQ e  MP = PR , teremos S pertencente
a BD e  V pertencente a AC.

8º) Como BP é bissetriz e intersecta AC no ponto médio, temos que AB=AC e
BP é perpendicular a BC.

9º) da mesma forma ACD é isósceles.

10º) ídem para BCD e ABD .


Conclusão : ABCD é um losango.. UFA .


Abraços

Carlos  Victor


Em 30 de outubro de 2014 12:22, Esdras Muniz esdrasmunizm...@gmail.com
escreveu:

 Opa, eu tinha entendido círculos circunscritos... Foi mal.

 Em 30 de outubro de 2014 11:02, Esdras Muniz esdrasmunizm...@gmail.com
 escreveu:



 Em 29 de outubro de 2014 22:50, Douglas Oliveira de Lima 
 profdouglaso.del...@gmail.com escreveu:


  *PROBLEMA 1 *

 Seja *ABCD *um quadrilátero convexo e seja *P *a interseção das
 diagonais *AC *e *BD*. Os raios dos círculos inscritos nos triângulos
 *ABP*, *BCP*, *CDP *e *DAP *são iguais. Prove que *ABCD *é um losango.


 Como poderíamos fazer esse problema?




 --
 Esta mensagem foi verificada pelo sistema de antivírus e
 acredita-se estar livre de perigo.




 --
 Esdras Muniz Mota
 Graduando em Matemática Bacharelado
 Universidade Federal do Ceará





 --
 Esdras Muniz Mota
 Graduando em Matemática Bacharelado
 Universidade Federal do Ceará



 --
 Esta mensagem foi verificada pelo sistema de antivírus e
 acredita-se estar livre de perigo.


-- 
Esta mensagem foi verificada pelo sistema de antiv�rus e
 acredita-se estar livre de perigo.



Re: [obm-l] Geometria.

2014-11-03 Por tôpico Carlos Victor
Oi  Pedro, esse  é um problema bem difícil e a solução, o Gandhi ( Antonio
Luis) me mostrou um tempo atrás ( 1997 se não me engano...). Vou tentar
escrevê-lo. Faça uma figura e acompanhe, ok ?

Vamos lá :

Vamos escolher dois pontos  M e N sobre BC, tais que N seja o simétrico de
E( ângulo em E igual a 18) em relação à bissetriz CF  e M o simétrico  de F
em relação à bissetriz  BE. Trace NI, NF  e trace ME.  Seja Q o encontro de
ME com FC. Conclua que os ângulos MQF, MQN e NQC são iguais a 60º. Como FN
é bissetriz do ângulo MFQ e NQ é bissetriz externa do ângulo MQF, temos
 que MN é bissetriz externa do ângulo QMF e daí encontre o ângulo interno
em B igual a 72º. Como o ângulo em A é 96º , temos que o ângulo interno em
C é igual 12º.

Donde  B- C = 60º, UFA !!!.

Caso não entenda alguma parte , escreva, ok ?


Abraços

Carlos  Victor

Em 3 de novembro de 2014 13:37, Pedro José petroc...@gmail.com escreveu:

 Seja ABC um triângulo e E e F os pés das bissetrizes internas dos ângulos
 B e C respectivamente. Sabendo-se que os ângulos E e F do triângulo EIF,
 onde I é o incentro de ABC, medem 18 e 24 graus, calcule B-C.

 Alguém tem alguma ideia?

 Grato,
 PJMS

 --
 Esta mensagem foi verificada pelo sistema de antivírus e
 acredita-se estar livre de perigo.

-- 
Esta mensagem foi verificada pelo sistema de antiv�rus e
 acredita-se estar livre de perigo.



Re: [obm-l] Problema de Geometria

2014-11-03 Por tôpico Carlos Victor
Oi Mariana,

Seja   x o  ângulo DCA . Aplicando a lei dos senos nos triângulos ACD e BCD
, vc encontrará a seguinte relação :

senx = 2sen(x+20).cos80.

Transformando em soma teremos : senx = sen(x+100) + sen(x-60).

Jogando para a esquerda o sen(x-60), teremos  senx - sen(x-60) =
sen(x+100); ou  seja :

2sen(30)cos(x-30) = sen(x+100) ; ou seja ;  sen(120-x) = sen(x+100) ; ou
seja :

x = 10º. Confira as contas, ok ?

Abraços

Carlos  Victor


PS : este problema se torna mais interessante, colocando o seguinte
enunciado :

No triângulo ABC, AB = AC . Um ponto D está sobre o lado AB e
AD = BC,  tal que  CD passe pelo circuncentro de ABC. Calcule os ângulos do
 triângulo.

Em 3 de novembro de 2014 20:21, Mariana Groff 
bigolingroff.mari...@gmail.com escreveu:

 Boa Noite,
 Alguém poderia me ajudar no problema a seguir?

 No triângulo ABC, AB = AC e BÂC = 20º. Um ponto D está sobre o lado AB e
 AD = BC. Calcule o ângulo BCD.

 Obrigada,
 Mariana

 --
 Esta mensagem foi verificada pelo sistema de antivírus e
 acredita-se estar livre de perigo.

-- 
Esta mensagem foi verificada pelo sistema de antiv�rus e
 acredita-se estar livre de perigo.



Re: [obm-l] Geometria

2014-10-25 Por tôpico Carlos Victor
Oi  Douglas,

Pense assim :

1) Mostre inicialmente que aplicando a lei dos senos para o triângulo OHA,
encontramos

cosB =2cosA.cosC., sabendo que AH = 2. OS, onde  S é o ponto médio de CB.

2) Sabendo que os lados do triângulo órtico são dados por :

Rsen2A, Rsen2B e Rsen2C e fazendo a semi-soma do primeiro com o último, e
utilizando (1),

conclua que esse triângulos tem as medidas dos lados em PA, ok ?

Nota : R é o raio do círculo circunscrito ao triângulo. ( confira as contas)


Abraços

Carlos Victor




Em 24 de outubro de 2014 07:07, Douglas Oliveira de Lima 
profdouglaso.del...@gmail.com escreveu:

 Bom dia a todos, não vi solução para essa questão,

 Sejam H, O o ortocentro e o circuncentro do triÂngulo ABC. AD, BE e CF são
 as alturas relativas aos vértices A, B e C. Suponha que OH seja paralelo
 a AC. Mostre que os lados do triângulo DEF estão em progressão aritmética.


 Agradeço a ajuda!!

 Douglas Oliveira.




 --
 Esta mensagem foi verificada pelo sistema de antivírus e
 acredita-se estar livre de perigo.

-- 
Esta mensagem foi verificada pelo sistema de antiv�rus e
 acredita-se estar livre de perigo.



Re: [obm-l] Inteiros

2014-09-13 Por tôpico Carlos Victor
Oi  Marcone, essa é do Mathematical Morsels.

Já que 3abc é positivo, devemos ter  a^3 maior que b^3  e c^3.

Logo ba  e ca dando b+c  2a  e portanto  a^2  4a , ou seja, a  4.

A segunda igualdade mostra também a é par , então  a = 2, b = c = 1.

Abraços

Carlos Victor

Em 13 de setembro de 2014 20:12, marcone augusto araújo borges 
marconeborge...@hotmail.com escreveu:

 Determine todos os naturais a,b e c tais que a^3 - b^3 - c^3 = 3abc e a^2
 = 2(b+c)

 --
 Esta mensagem foi verificada pelo sistema de antivírus e
 acredita-se estar livre de perigo.


-- 
Esta mensagem foi verificada pelo sistema de antiv�rus e
 acredita-se estar livre de perigo.



Re: [obm-l] Problema de geometria!

2014-07-06 Por tôpico Carlos Victor
OI Douglas ,

Pensando neste problema, se usar a lei dos cossenos nos triângulos FCE, AFD
e DBE e usando o fato de que cos(90+B)= -senB ( não é muito trabalhoso);
 deixando na forma de quadrados não é difícil de concluir que 4 FE^2= ED^2
e que 4DF^2 = 3ED^2 ; ou seja o triângulo EFD é retângulo e que que os
ângulos pedidos são 90º e 30º , ok ?

Pelo visto vc está querendo uma solução com algum traçado mágico, não é
verdade?
Estarei pensando, ok  Douglas! Estou desconfiado que deve ter alguma coisa
com o Teorema de Napoleão ...

Vamos tentar, pois deve ser assaz interessante tal traçado.

Abraços

Carlos Victor


Em 13 de junho de 2014 16:09, Douglas Oliveira de Lima 
profdouglaso.del...@gmail.com escreveu:

 Olá caros amigos, me encontro mais uma vez com um pequeno problema de
 geometria no qual estou com uma solução muito absurda(muito trabalho
 braçal), gostaria de uma ajuda com outras soluções, desde já agradeço a
 colaboração dos senhores.

 PROBLEMA:
 Considere um triângulo ABC, são construídos externamente os triângulos ADB
 e BCE de forma que ADB=BEC=90 GRAUS E DAB=EBC=30 graus. No segmento AC
 marca-se o ponto F tal que AF=3FC. Calcular os ângulos DFE e FDE.

 Douglas Oliveira de Lima

 --
 Esta mensagem foi verificada pelo sistema de antivírus e
 acredita-se estar livre de perigo.

-- 
Esta mensagem foi verificada pelo sistema de antiv�rus e
 acredita-se estar livre de perigo.



Re: [obm-l] Provar perpendicularidade em Geometria Plana

2014-05-25 Por tôpico Carlos Victor
Oi Martins, Observe o seguinte :

Os segmentos AE e AF são respectivamente : c.cosA e bcosA.
Observe agora que os triângulos ABC e AEF são semelhantes, por possuirem
 os lados AC e AB  com razões iguais aos lados AE e AF e, claro um ângulo
em comum.

Donde o ângulo FEA = ângulo em B.

 Como o ângulo OAC = 90 - B, teremos o
ângulo ERA = 90 graus, ok ?

Abraços

Victor


Em 25 de maio de 2014 11:03, Martins Rama martin...@pop.com.br escreveu:

 Caros amigos, alguém me auxilia nessa demonstração de Geom Plana? É do
 livro da SBM do Antonio Caminha Muniz Neto.
 Abraço a todos.
 Martins Rama.

 Seja ABC um triângulo acutângulo de circuncentro O. Se E e F são os pés
 das alturas relativas aos vértices B e C, respectivamente, prove que o
 segmento OA é perpendicular ao segmento EF.
 --
 Esta mensagem foi verificada pelo sistema de antivírus e
 acredita-se estar livre de perigo.


-- 
Esta mensagem foi verificada pelo sistema de antiv�rus e
 acredita-se estar livre de perigo.



Re: [obm-l] Provar perpendicularidade em Geometria Plana

2014-05-25 Por tôpico Carlos Victor
Oi Martins, esqueci de dizer que o ponto  R é a interseccão de OA e EF, ok ?

Abraços
Carlos Victor


Em 25 de maio de 2014 13:31, Carlos Victor victorcar...@globo.comescreveu:

 Oi Martins, Observe o seguinte :

 Os segmentos AE e AF são respectivamente : c.cosA e bcosA.
 Observe agora que os triângulos ABC e AEF são semelhantes, por possuirem
  os lados AC e AB  com razões iguais aos lados AE e AF e, claro um ângulo
 em comum.

 Donde o ângulo FEA = ângulo em B.

  Como o ângulo OAC = 90 - B, teremos o
 ângulo ERA = 90 graus, ok ?

 Abraços

 Victor


 Em 25 de maio de 2014 11:03, Martins Rama martin...@pop.com.br escreveu:

 Caros amigos, alguém me auxilia nessa demonstração de Geom Plana? É do
 livro da SBM do Antonio Caminha Muniz Neto.
 Abraço a todos.
 Martins Rama.

 Seja ABC um triângulo acutângulo de circuncentro O. Se E e F são os pés
 das alturas relativas aos vértices B e C, respectivamente, prove que o
 segmento OA é perpendicular ao segmento EF.
 --
 Esta mensagem foi verificada pelo sistema de antivírus e
 acredita-se estar livre de perigo.




-- 
Esta mensagem foi verificada pelo sistema de antiv�rus e
 acredita-se estar livre de perigo.



Re: [obm-l] Provar perpendicularidade em Geometria Plana

2014-05-25 Por tôpico Carlos Victor
Oi Hermann, certamente a sua solução é bonita e mais simples.

Abraços

Carlos Victor


Em 25 de maio de 2014 16:02, Hermann ilhadepaqu...@bol.com.br escreveu:

 A solução do Carlos é excelente.

 A minha solução é só com arco capaz

 Como o ângulo BEC e BFC =90 temos a circunferência BEFC e nela observamos
 pelo arco FC que os ângulos B e E são iguais.

 Agora (como já visto pelo Carlos) na circunferência BAC o ângulo A = 90 - B


 E o  segmento OA é perpendicular ao segmento EF.

 Abraços
 Hermann



 - Original Message - From: Martins Rama
 To: OBM-L
 Sent: Sunday, May 25, 2014 11:03 AM
 Subject: [obm-l] Provar perpendicularidade em Geometria Plana



 Caros amigos, alguém me auxilia nessa demonstração de Geom Plana? É do
 livro da SBM do Antonio Caminha Muniz Neto.
 Abraço a todos.
 Martins Rama.

 Seja ABC um triângulo acutângulo de circuncentro O. Se E e F são os pés
 das alturas relativas aos vértices B e C, respectivamente, prove que o
 segmento OA é perpendicular ao segmento EF.
 --
 Esta mensagem foi verificada pelo sistema de antiv�rus e

 acredita-se estar livre de perigo.

 --
 Esta mensagem foi verificada pelo sistema de antivírus e
 acredita-se estar livre de perigo.

 =
 Instru�ões para entrar na lista, sair da lista e usar a lista em
 http://www.mat.puc-rio.br/~obmlistas/obm-l.html
 =


-- 
Esta mensagem foi verificada pelo sistema de antiv�rus e
 acredita-se estar livre de perigo.



Re: [obm-l] Re: Mais uma que quero compartilhar!!

2014-05-17 Por tôpico Carlos Victor
Esse é um dos problemas mais lindo que o  meu grande companheiro Gandhi me
apresentou.
Abraços Douglas.

Carlos Victor


Em 15 de maio de 2014 23:16, Douglas Oliveira de Lima 
profdouglaso.del...@gmail.com escreveu:

 Eu nao sei se deu pra compreender direito a expressão , mas acho que
 escrevi certinho, o resultado da 3.



 Em 15 de maio de 2014 17:45, Douglas Oliveira de Lima 
 profdouglaso.del...@gmail.com escreveu:

 Essa vai em homenagem a meu grande amigo Gandhi(Antonio Luiz Santos) que
 me ensinou como fazer, quero dizer também que essa lista da obm(do qual
 usamos para discutir questões de olimpíadas e outras questões
 interessantes) esta sendo pra mim muito gratificante neste momento, porque
 nos que gostamos de matemática, gostamos de resolver questões ate em papel
 de guardanapo no restaurante,pois desde que me mudei para Brasilia não
 encontrei professores aqui como os que tive a oportunidade de  conhecer no
 Rio de Janeiro como Gandhi, Carlos Victor, Eduardo Mauro, Eduardo Wagner,
 Haroldo, Poncio, Ivan, Bandeira, e claro não poderia esquecer do grande
 Alvaro, e que me ajudaram a crescer dentro desta área. Existem outros que
 conheci, mas hoje o mérito vai pra eles,professores humildes, que nunca me
 negaram sequer uma questão, ajudando o amigo a crescer para que um dia
 possamos derrubar essa grande massa de professores ruins(que não gostam de
 estudar) do mercado,e quem sabe assim incentivando a futura geração a
 curtir matemática.

 Encontrar o valor de (1+2(1+3(1+4(1+...)^(1/2))^(1/2))^(1/2))^(1/2)



 --
 Esta mensagem foi verificada pelo sistema de antivírus e
 acredita-se estar livre de perigo.


-- 
Esta mensagem foi verificada pelo sistema de antiv�rus e
 acredita-se estar livre de perigo.



[obm-l] Re: [obm-l] Retângulo de incentros num quadrilatero inscritível

2014-04-05 Por tôpico Carlos Victor
Olá Martins,

Vamos mostrar inicialmente que o ângulo (abc) é reto, então para os outros
 a demonstração é idêntica.

1) Como ABCD é inscritível, então os ângulos ADB e ACB são iguais. Da mesma
forma  que os ângulos ABD e DCA são iguais, ok ?

2) Vamos denominar o seguinte :

DCA = ABD = 2x ; CAD= DBC=2z ; ADB = ACB = 2y ; BDC= 2t .

3) Observe que o ângulo DbC = 90+z = DaC ; donde se conclui que o
quadrilátero abDC é inscritível, ok ?
Logo os ângulos Cba e  CDa são iguais, ou seja, Cba= t.

4) Pelo mesmo motivo, os ângulos  Dcb = DAb= z   e ADc = Abc = y.

5) Agora, olhando para o vértice b  e tomando  a soma dos ângulos  em
torno dele  igual a 360 graus e  fazendo x+y+z +t =90 graus, encontraremos
o ângulo b = 90 graus , do quadrilátero abcd.

Para os outros vértices , utilizamos a mesma ideia.

Abraços

Carlos  Victor


Em 4 de abril de 2014 15:59, martin...@pop.com.br escreveu:

 Olá amigos da lista. Não estou conseguindo resolver esta de Geometria
 Plana:

 Seja ABCD um quadrilátero inscritível e sejam I(a), I(b), I(c) e I(d),
 respectivamente, os incentros dos triângulos BCD, CDA, DAB e ABC. Prove que
 o quadrilátero I(a)I(b)I(c)I(d) é um retângulo.

 Alguém pode ajudar?

 Martins Rama.


 --
 Esta mensagem foi verificada pelo sistema de antivírus e
 acredita-se estar livre de perigo.


-- 
Esta mensagem foi verificada pelo sistema de antivírus e
 acredita-se estar livre de perigo.



[obm-l] Re: [obm-l] Retângulo de incentros num quadrilatero inscritível

2014-04-05 Por tôpico Carlos Victor
No email anterior, onde estiver a , lê-se I(a)...; ok ?

Abraços

Carlos Victor


Em 5 de abril de 2014 14:27, Carlos Victor victorcar...@globo.comescreveu:

 Olá Martins,

 Vamos mostrar inicialmente que o ângulo (abc) é reto, então para os outros
  a demonstração é idêntica.

 1) Como ABCD é inscritível, então os ângulos ADB e ACB são iguais. Da
 mesma forma  que os ângulos ABD e DCA são iguais, ok ?

 2) Vamos denominar o seguinte :

 DCA = ABD = 2x ; CAD= DBC=2z ; ADB = ACB = 2y ; BDC= 2t .

 3) Observe que o ângulo DbC = 90+z = DaC ; donde se conclui que o
 quadrilátero abDC é inscritível, ok ?
 Logo os ângulos Cba e  CDa são iguais, ou seja, Cba= t.

 4) Pelo mesmo motivo, os ângulos  Dcb = DAb= z   e ADc = Abc = y.

 5) Agora, olhando para o vértice b  e tomando  a soma dos ângulos  em
 torno dele  igual a 360 graus e  fazendo x+y+z +t =90 graus, encontraremos
 o ângulo b = 90 graus , do quadrilátero abcd.

 Para os outros vértices , utilizamos a mesma ideia.

 Abraços

 Carlos  Victor


 Em 4 de abril de 2014 15:59, martin...@pop.com.br escreveu:

 Olá amigos da lista. Não estou conseguindo resolver esta de Geometria
 Plana:

 Seja ABCD um quadrilátero inscritível e sejam I(a), I(b), I(c) e I(d),
 respectivamente, os incentros dos triângulos BCD, CDA, DAB e ABC. Prove que
 o quadrilátero I(a)I(b)I(c)I(d) é um retângulo.

 Alguém pode ajudar?

 Martins Rama.


 --
 Esta mensagem foi verificada pelo sistema de antivírus e
 acredita-se estar livre de perigo.




-- 
Esta mensagem foi verificada pelo sistema de antivírus e
 acredita-se estar livre de perigo.



[obm-l] Re: [obm-l] Re: [obm-l] Re: [obm-l] Re: [obm-l] Re: [obm-l] Re: [obm-l] Re: [obm-l] Re: [obm-l] RE: [obm-l] Re: [obm-l] Três de inteiros

2014-02-24 Por tôpico Carlos Victor
Pacini,

 vc tem que  retirar os casos de que  x+y+z =0 , ok ?

Carlos Victor


Em 24 de fevereiro de 2014 16:44, Pacini Bores pacini.bo...@globo.comescreveu:

 Olá pessoal, posso fazer o que está descrito a seguir  no terceiro
 problema ?

 Sabemos que x^2+y^2+z^2 ** xy+xz+yz  e na hipótese de que  xy+xz+yz não
 seja nulo, teremos :

 (x^2 + y^2 + z^2)/2(xy+yz+xz) ** 1/2 , para xy+xz+yz   0   e

 (x^2 + y^2 + z^2)/2(xy+yz+xz) ** -1/2 , para xy+xz+yz   0 .

 Daí  F(x,y,z)  varia de  [-1/2, 0[  união  [1/2,+infinito[ .

 Pacini





 Em 24 de fevereiro de 2014 12:43, terence thirteen 
 peterdirich...@gmail.com escreveu:

 Quanto ao último,

 3) Se x,y,z são números reais não nulos,com x+y+z também não nulo
 Calcule os valores possíveis da expressão F(x,y,z) = (x^2 + y^2 +
 z^2)/2(xy+yz+xz)

 Acho que dá para aplicar rearranjo, não?

 Primeiro, por homogeneidade, supunhetemos que x+y+z=1. Segundo, por
 simetria, x=y=z.

 Temos x^2+y^2+z^2 = x^2+yz+zy, afinal basta subtrair:

 (x^2-x^2) + (y^2-yz) + (z^2-zy) = 0 + y(y-z) + z(z-y) = (z-y)^2 =0

 E também,

 x^2+yz+zy = xy+yz+zy

 Demonstre da mesma forma!

 Agora, temos que ver os sinais...

 Em 21/02/14, Tarsis Esautarsise...@gmail.com escreveu:
  Bernado, vc tinha razão. resolvendo 2) a resposta é (-33, -33).
 
  Desenvolvendo 2) (m + n)² + (m + n).33 + 33² = 3mn, vamos chegar a
 
  m² -mn + 33m + n² + 33n + 33² = 0
 
  Resolvendo em função de n, teremos um delta [(n-33)² - 4.(n² + 33n +
 33²)]
  = -3n² -6.33n - 3.33²,
 
  Sendo que -3n² -6.33n - 3.33² =0
 
  Estudando o sinal da parábola, temos que a concavidade deve estar
 voltada
  para baixo, assumindo assim um máximo
 
  O delta desta nova equação é 0 e ela apresenta máximo em n = -33.
  Substituindo-se em 2), m = -33.
 
 
 
 
 
 
 
  2014-02-21 17:02 GMT-03:00 Bernardo Freitas Paulo da Costa 
  bernardo...@gmail.com:
 
  2014-02-21 14:24 GMT-03:00 Tarsis Esau tarsise...@gmail.com:
   Fiz a segunda, vou tentar fazer a terceira :)
  
   m³ + n³ + 99mn = 33³
  
   (m + n)³ - 3m²n - 3mn² + 99mn = 33³
   (m + n)³ - 33³ = 3mn.[(m + n) - 33]
   [(m +n) - 33].[(m + n)² + (m +n).33 + 33²] = 3mn.[(m+n) - 33]
  
   Assim, temos
  
   1) m + n - 33 = 0
  
   e
 
  Deveria ser ou, mas você agiu como se fosse ou. Mas isso é menos
  importante que o meu próximo comentário.
 
   2) (m + n)² + (m + n).33 + 33² = 3mn
  
   De 1) temos todos os pares (x,y): (0,33); (1,32), ..., (32, 1), (33,
   0).
   Todos os inteiros estão neste intervalo.
  
   Uma vez que , caso um m seja maior que 33, o n necessariamente deve
 ser
   menor que zero, o que vai contra o enunciado de m.n =0.
  
   Desse modo, não há necessidade de resolver 2).
 
  Claro que há. Pode ser que a equação 2 tenha uma solução com m = 20 e
  n = 10 (sei lá) cuja soma não é 33. Se você tivesse obtido TODOS os
  pares (a,b) com 0=a=33, 0 =b=33 como solução, aí tava certo. Mas
  veja que essa é uma equação cúbica, portanto para cada m existem
  três soluções n possíveis. É bastante provável que, se m é inteiro,
  não haja muitas soluções com n inteiro, mas você tem que demonstrar
  isso. Além disso, o enunciado diz que m.n = 0, ou seja, pode ser que
  m e n sejam NEGATIVOS! (mas talvez o enunciado tenha sido copiado
  errado, e era para ser m E n = 0).
  --
  Bernardo Freitas Paulo da Costa
 
  --
  Esta mensagem foi verificada pelo sistema de antivírus e
   acredita-se estar livre de perigo.
 
 
 
 =
  Instruções para entrar na lista, sair da lista e usar a lista em
  http://www.mat.puc-rio.br/~obmlistas/obm-l.html
 
 =
 
 
  --
  Esta mensagem foi verificada pelo sistema de antivírus e
   acredita-se estar livre de perigo.
 
 


 --
 /**/
 神が祝福

 Torres

 --
 Esta mensagem foi verificada pelo sistema de antivírus e
  acredita-se estar livre de perigo.


 =
 Instru�ões para entrar na lista, sair da lista e usar a lista em
 http://www.mat.puc-rio.br/~obmlistas/obm-l.html
 =



 --
 Esta mensagem foi verificada pelo sistema de antivírus e
 acredita-se estar livre de perigo.


-- 
Esta mensagem foi verificada pelo sistema de antiv�rus e
 acredita-se estar livre de perigo.



Re: [obm-l] R^2=(BC^2+AH^2)/4

2014-02-18 Por tôpico Carlos Victor
Olá luís,

O enunciado é este com o seguinte detalhe : H é o ortocentro de ABC, daí AH
não é a altura, ok ?

Abraços

Victor


Em 17 de fevereiro de 2014 23:29, luiz silva
luizfelipec...@yahoo.com.brescreveu:

 É porque eu não vi o enunciado. Seria assim : Em qualquer triângulo ABC, a
 soma do quadrado do lado BC e do quadrado da altura AH, relativa a BC, é
 igual ao quadrado do diâmetro do círculo circunscrito?

 Abs
 Felipe


   Em Segunda-feira, 17 de Fevereiro de 2014 19:02, Carlos Victor 
 victorcar...@globo.com escreveu:
  Sim Luís,
 Você pode encontrar essa relação em vários livros de geometria que fale
 sobre a reta de Euler,   que passa  pelo circuncentro, ortocentro e
 baricentro, ok ?
 Abraços

 Carlos Victor


 Em 17 de fevereiro de 2014 18:33, luiz silva 
 luizfelipec...@yahoo.com.brescreveu:

 Essa relação é valida em um triangulo qualquer ?

 Abs
 Felipe



   Em Segunda-feira, 17 de Fevereiro de 2014 15:49, Carlos Victor 
 victorcar...@globo.com escreveu:
   Oi Luís,
 Apesar do enunciado não falar, H é o ortocentro do triângulo, ok ?

 Abraços

 Carlos Victor


 Em 16 de fevereiro de 2014 22:33, luiz silva 
 luizfelipec...@yahoo.com.brescreveu:

 AH é a altura relativa à BC?


   Em Sábado, 15 de Fevereiro de 2014 17:30, Carlos Victor 
 victorcar...@globo.com escreveu:
  Oi Luís,
 digitei errado.

 Onde está AM lê-se  AH, ok ?

 Desculpe  o engano...

 Carlos  Victor


 Em 15 de fevereiro de 2014 16:53, Carlos Victor 
 victorcar...@globo.comescreveu:

 Oi Luís,

 Seja M o ponto médio de BC  e O o circuncentro do triângulo ABC. Prove
 inicialmente que AM= 2.OM http://2.om/ e aplique Pitágoras no triângulo
 OMC, por exemplo.
 Daí sai legal a relação que tu queres, ok ?

 Para provar que AM = 2.OM http://2.om/ , pense no alinhamento que
 existe entre o circuncentro, ortocentro e baricentro... .

 Abraços

 Carlos  Victor


 Em 13 de fevereiro de 2014 13:13, Luís qed_te...@hotmail.com escreveu:

  Sauda,c~oes,

 Como provar a relação abaixo?

 R^2=(BC^2+AH^2)/4

 Imaginei colocar os pontos B,C,H com as seguintes coordenadas:

 B=(0,0)  C=(a,0)  H=(h,y_H) A=(h,y_A)

 Daí a gente obtém o ponto H_c=(x,y) com régua e compasso e
 em seguida o ponto A. O circuncentro (O) é calculado e finalmente R.

 As contas não são legais com papel e lápis. Alguém poderia dar as
 coordenadas dos pontos A e (O) usando um programa de
 cálculo simbólico ?

 Obrigado.

 Luís


 --
 Esta mensagem foi verificada pelo sistema de antivírus e
 acredita-se estar livre de perigo.




 --
 Esta mensagem foi verificada pelo sistema de antivírus e
 acredita-se estar livre de perigo.



 --
 Esta mensagem foi verificada pelo sistema de antivírus e
 acredita-se estar livre de perigo.



 --
 Esta mensagem foi verificada pelo sistema de antivírus e
 acredita-se estar livre de perigo.



 --
 Esta mensagem foi verificada pelo sistema de antivírus e
 acredita-se estar livre de perigo.



 --
 Esta mensagem foi verificada pelo sistema de antivírus e
 acredita-se estar livre de perigo.



 --
 Esta mensagem foi verificada pelo sistema de antivírus e
 acredita-se estar livre de perigo.


-- 
Esta mensagem foi verificada pelo sistema de antivírus e
 acredita-se estar livre de perigo.



Re: [obm-l] R^2=(BC^2+AH^2)/4

2014-02-17 Por tôpico Carlos Victor
Oi Luís,
Apesar do enunciado não falar, H é o ortocentro do triângulo, ok ?

Abraços

Carlos Victor


Em 16 de fevereiro de 2014 22:33, luiz silva
luizfelipec...@yahoo.com.brescreveu:

 AH é a altura relativa à BC?


   Em Sábado, 15 de Fevereiro de 2014 17:30, Carlos Victor 
 victorcar...@globo.com escreveu:
  Oi Luís,
 digitei errado.

 Onde está AM lê-se  AH, ok ?

 Desculpe  o engano...

 Carlos  Victor


 Em 15 de fevereiro de 2014 16:53, Carlos Victor 
 victorcar...@globo.comescreveu:

 Oi Luís,

 Seja M o ponto médio de BC  e O o circuncentro do triângulo ABC. Prove
 inicialmente que AM= 2.OM http://2.om/ e aplique Pitágoras no triângulo
 OMC, por exemplo.
 Daí sai legal a relação que tu queres, ok ?

 Para provar que AM = 2.OM http://2.om/ , pense no alinhamento que
 existe entre o circuncentro, ortocentro e baricentro... .

 Abraços

 Carlos  Victor


 Em 13 de fevereiro de 2014 13:13, Luís qed_te...@hotmail.com escreveu:

  Sauda,c~oes,

 Como provar a relação abaixo?

 R^2=(BC^2+AH^2)/4

 Imaginei colocar os pontos B,C,H com as seguintes coordenadas:

 B=(0,0)  C=(a,0)  H=(h,y_H) A=(h,y_A)

 Daí a gente obtém o ponto H_c=(x,y) com régua e compasso e
 em seguida o ponto A. O circuncentro (O) é calculado e finalmente R.

 As contas não são legais com papel e lápis. Alguém poderia dar as
 coordenadas dos pontos A e (O) usando um programa de
 cálculo simbólico ?

 Obrigado.

 Luís


 --
 Esta mensagem foi verificada pelo sistema de antivírus e
 acredita-se estar livre de perigo.




 --
 Esta mensagem foi verificada pelo sistema de antivírus e
 acredita-se estar livre de perigo.



 --
 Esta mensagem foi verificada pelo sistema de antivírus e
 acredita-se estar livre de perigo.


-- 
Esta mensagem foi verificada pelo sistema de antivírus e
 acredita-se estar livre de perigo.



Re: [obm-l] R^2=(BC^2+AH^2)/4

2014-02-17 Por tôpico Carlos Victor
Sim Luís,
Você pode encontrar essa relação em vários livros de geometria que fale
sobre a reta de Euler,   que passa  pelo circuncentro, ortocentro e
baricentro, ok ?
Abraços

Carlos Victor


Em 17 de fevereiro de 2014 18:33, luiz silva
luizfelipec...@yahoo.com.brescreveu:

 Essa relação é valida em um triangulo qualquer ?

 Abs
 Felipe



   Em Segunda-feira, 17 de Fevereiro de 2014 15:49, Carlos Victor 
 victorcar...@globo.com escreveu:
  Oi Luís,
 Apesar do enunciado não falar, H é o ortocentro do triângulo, ok ?

 Abraços

 Carlos Victor


 Em 16 de fevereiro de 2014 22:33, luiz silva 
 luizfelipec...@yahoo.com.brescreveu:

 AH é a altura relativa à BC?


   Em Sábado, 15 de Fevereiro de 2014 17:30, Carlos Victor 
 victorcar...@globo.com escreveu:
  Oi Luís,
 digitei errado.

 Onde está AM lê-se  AH, ok ?

 Desculpe  o engano...

 Carlos  Victor


 Em 15 de fevereiro de 2014 16:53, Carlos Victor 
 victorcar...@globo.comescreveu:

 Oi Luís,

 Seja M o ponto médio de BC  e O o circuncentro do triângulo ABC. Prove
 inicialmente que AM= 2.OM http://2.om/ e aplique Pitágoras no triângulo
 OMC, por exemplo.
 Daí sai legal a relação que tu queres, ok ?

 Para provar que AM = 2.OM http://2.om/ , pense no alinhamento que
 existe entre o circuncentro, ortocentro e baricentro... .

 Abraços

 Carlos  Victor


 Em 13 de fevereiro de 2014 13:13, Luís qed_te...@hotmail.com escreveu:

  Sauda,c~oes,

 Como provar a relação abaixo?

 R^2=(BC^2+AH^2)/4

 Imaginei colocar os pontos B,C,H com as seguintes coordenadas:

 B=(0,0)  C=(a,0)  H=(h,y_H) A=(h,y_A)

 Daí a gente obtém o ponto H_c=(x,y) com régua e compasso e
 em seguida o ponto A. O circuncentro (O) é calculado e finalmente R.

 As contas não são legais com papel e lápis. Alguém poderia dar as
 coordenadas dos pontos A e (O) usando um programa de
 cálculo simbólico ?

 Obrigado.

 Luís


 --
 Esta mensagem foi verificada pelo sistema de antivírus e
 acredita-se estar livre de perigo.




 --
 Esta mensagem foi verificada pelo sistema de antivírus e
 acredita-se estar livre de perigo.



 --
 Esta mensagem foi verificada pelo sistema de antivírus e
 acredita-se estar livre de perigo.



 --
 Esta mensagem foi verificada pelo sistema de antivírus e
 acredita-se estar livre de perigo.



 --
 Esta mensagem foi verificada pelo sistema de antivírus e
 acredita-se estar livre de perigo.


-- 
Esta mensagem foi verificada pelo sistema de antivírus e
 acredita-se estar livre de perigo.



Re: [obm-l] R^2=(BC^2+AH^2)/4

2014-02-15 Por tôpico Carlos Victor
Oi Luís,

Seja M o ponto médio de BC  e O o circuncentro do triângulo ABC. Prove
inicialmente que AM= 2.OM e aplique Pitágoras no triângulo OMC, por exemplo.
Daí sai legal a relação que tu queres, ok ?

Para provar que AM = 2.OM , pense no alinhamento que existe entre o
circuncentro, ortocentro e baricentro... .

Abraços

Carlos  Victor


Em 13 de fevereiro de 2014 13:13, Luís qed_te...@hotmail.com escreveu:

 Sauda,c~oes,

 Como provar a relação abaixo?

 R^2=(BC^2+AH^2)/4

 Imaginei colocar os pontos B,C,H com as seguintes coordenadas:

 B=(0,0)  C=(a,0)  H=(h,y_H) A=(h,y_A)

 Daí a gente obtém o ponto H_c=(x,y) com régua e compasso e
 em seguida o ponto A. O circuncentro (O) é calculado e finalmente R.

 As contas não são legais com papel e lápis. Alguém poderia dar as
 coordenadas dos pontos A e (O) usando um programa de
 cálculo simbólico ?

 Obrigado.

 Luís


 --
 Esta mensagem foi verificada pelo sistema de antivírus e
 acredita-se estar livre de perigo.


-- 
Esta mensagem foi verificada pelo sistema de antivírus e
 acredita-se estar livre de perigo.



Re: [obm-l] R^2=(BC^2+AH^2)/4

2014-02-15 Por tôpico Carlos Victor
Oi Luís,
digitei errado.

Onde está AM lê-se  AH, ok ?

Desculpe  o engano...

Carlos  Victor


Em 15 de fevereiro de 2014 16:53, Carlos Victor
victorcar...@globo.comescreveu:

 Oi Luís,

 Seja M o ponto médio de BC  e O o circuncentro do triângulo ABC. Prove
 inicialmente que AM= 2.OM e aplique Pitágoras no triângulo OMC, por
 exemplo.
 Daí sai legal a relação que tu queres, ok ?

 Para provar que AM = 2.OM , pense no alinhamento que existe entre o
 circuncentro, ortocentro e baricentro... .

 Abraços

 Carlos  Victor


 Em 13 de fevereiro de 2014 13:13, Luís qed_te...@hotmail.com escreveu:

  Sauda,c~oes,

 Como provar a relação abaixo?

 R^2=(BC^2+AH^2)/4

 Imaginei colocar os pontos B,C,H com as seguintes coordenadas:

 B=(0,0)  C=(a,0)  H=(h,y_H) A=(h,y_A)

 Daí a gente obtém o ponto H_c=(x,y) com régua e compasso e
 em seguida o ponto A. O circuncentro (O) é calculado e finalmente R.

 As contas não são legais com papel e lápis. Alguém poderia dar as
 coordenadas dos pontos A e (O) usando um programa de
 cálculo simbólico ?

 Obrigado.

 Luís


 --
 Esta mensagem foi verificada pelo sistema de antivírus e
 acredita-se estar livre de perigo.




-- 
Esta mensagem foi verificada pelo sistema de antivírus e
 acredita-se estar livre de perigo.



Re: [obm-l] Apostila de Desenho 2 Impacto OFF TOPIC

2013-12-03 Por tôpico Carlos Victor
Obrigado João,

Envie-me a sua conta bancária para depósito, ok ?

Também posso lhe enviar o custo pelo correio.

Agradeço e fico a disposição para o que precisares.

Abraços

Carlos Victor


Em 1 de dezembro de 2013 17:54, jjun...@fazenda.ms.gov.br escreveu:

  Senhores:

 Ontem (sábado), por volta das 15h em Campo Grande, foi enviada cópia da
 Apostila 2 de Desenho do IMPACTO, uma ao senhor Carlos Victor (Nilópolis -
 RJ), e outra a Graciliano Antônio Damazo (Penápolis - SP).

 ATT.
 João (Campo Grande - MS)

 --
 Esta mensagem foi verificada pelo sistema de antivírus e
 acredita-se estar livre de perigo.


-- 
Esta mensagem foi verificada pelo sistema de antivírus e
 acredita-se estar livre de perigo.



Re: [obm-l] Como que faz??

2013-09-27 Por tôpico Carlos Victor
Olá Eduardo ,
Observe que na circunferência  C´ , o arco CE é dado por : 2x+2y ; pois AC
é tangente  em  à C´ já que o ângulo externo em C no triângulo ACE é dado
por : x+ y . O ângulo CGE é inscrito na circunferência  C   , ok ? . Note
que o ângulo ADE = x+y , está inscrito na circunferência C´´ .

Abs
Carlos  Victor


Em 26 de setembro de 2013 01:05, Eduardo Wilner
eduardowil...@yahoo.com.brescreveu:

 Oi Carlos.

 No item 2) vc. diz que CGE = x+y; isto significaria,  CGE = ADE . Vc.
 poderia explicar?

 Obrigado

 [ ]'s


   --
  *De:* Carlos Victor victorcar...@globo.com
 *Para:* obm-l@mat.puc-rio.br
 *Enviadas:* Terça-feira, 24 de Setembro de 2013 19:30
 *Assunto:* Re: [obm-l] Como que faz??

 Olá Douglas,
 Acredito ter conseguido uma resolução para o problema 2 de geometria que
 vc postou aqui  .

 Vamos lá  e acompanhe fazendo a figura , ok ?
 vamos provar que na verdade o ângulo DEF é o dobro de ADC.
 Seja  o ângulo  ADC = x e o ângulo CDE = y .

 1) Trace CE e observe que o quadrilátero ACED é inscritível . então AEC =
 x  e  EAC = y .

 2) seja G a intersecção de CD com a circunferência C´ . Trace  EG e
 observe que  o ângulo CGE = x + y . Daí concluímos que o ângulo GED = x .

 3) Não é difícil de mostrar que EB  é bissetriz de AEG . Seja então os
 ângulos AEB= DEB = z .

 4) Trace agora a perpendicular de B  ao segmento ED e seja H o pé desta
 perpendicular. Observe que o quadrilátero BFHD é inscritível , então BHF =
 x .Trace FH e observe que EG é perpendicular  a FH . Seja J a intersecção
 de FH com EG .

 5) Como o triângulo CEG está inscrito na circunferência C´ e observando
 que BF é perpendicular ao lado CD , pelo enunciado ; teremos pela reta de
 SIMSON , que  os pés das  perpendiculares traçadas de B aos lados CG , EG e
 EC  estão alinhados. Sejam então I o pé da perpendicular traçada de ao lado
 EG  e R o pé  da perpendicular traçada de B ao lado CE .

 6) observando os quadriláteros inscritíveis : BIER , BIHE , teremos q









 Em 23 de agosto de 2013 16:03, douglas.olive...@grupoolimpo.com.brescreveu:

 **
 Olá , alguns alunos do ensino médio da instituição onde trabalho me deram
 alguns problemas do site https://brilliant.org/
 E não consegui achar solução para dois deles, vou escreve-los abaixo e se
 alguém puder me ajudar agradeço.

 PROBLEMA 1: Dada uma função f:R-R tal que f(2x^2 -1)=2(f(x))^2 -1 e f(x)
 é um polinômio de grau 13, sendo assim determine o coeficiente de x^5 de
 f(x).

 PROBLEMA 2: Seja uma circunferência C'  e um ponto externo A , traça-se
 por A duas tangentes a circunferência que a interceptam nos pontos B e C ,
 marca-se no prolongamento de AB no sentido de A para B um ponto D tal que o
 ângulo ADC=25 graus, traça-se por B uma perpendicular ao segmento CD que
 intercepta CD em F . Agora considere um outra circunferência C''
 circunscrita ao triângulo ADC que intercepta a primeira circunferência C'
 no ponto E . Determinar a medida do ângulo DEF.


 Obs: Fiz a segunda figura no geogebra e encontrei 50 graus como resposta ,
 preciso na verdade de uma resolução.

 Att, Douglas Oliveira.


 --
 Esta mensagem foi verificada pelo sistema de antivírus e
 acredita-se estar livre de perigo.



 --
 Esta mensagem foi verificada pelo sistema de antivírus e
 acredita-se estar livre de perigo.



 --
 Esta mensagem foi verificada pelo sistema de antivírus e
 acredita-se estar livre de perigo.


-- 
Esta mensagem foi verificada pelo sistema de antivírus e
 acredita-se estar livre de perigo.



Re: [obm-l] Como que faz??

2013-09-24 Por tôpico Carlos Victor
Olá Douglas,
Acredito ter conseguido uma resolução para o problema 2 de geometria que vc
postou aqui  .

Vamos lá  e acompanhe fazendo a figura , ok ?
vamos provar que na verdade o ângulo DEF é o dobro de ADC.
Seja  o ângulo  ADC = x e o ângulo CDE = y .

1) Trace CE e observe que o quadrilátero ACED é inscritível . então AEC = x
 e  EAC = y .

2) seja G a intersecção de CD com a circunferência C´ . Trace  EG e observe
que  o ângulo CGE = x + y . Daí concluímos que o ângulo GED = x .

3) Não é difícil de mostrar que EB  é bissetriz de AEG . Seja então os
ângulos AEB= DEB = z .

4) Trace agora a perpendicular de B  ao segmento ED e seja H o pé desta
perpendicular. Observe que o quadrilátero BFHD é inscritível , então BHF =
x .Trace FH e observe que EG é perpendicular  a FH . Seja J a intersecção
de FH com EG .

5) Como o triângulo CEG está inscrito na circunferência C´ e observando que
BF é perpendicular ao lado CD , pelo enunciado ; teremos pela reta de
SIMSON , que  os pés das  perpendiculares traçadas de B aos lados CG , EG e
EC  estão alinhados. Sejam então I o pé da perpendicular traçada de ao lado
EG  e R o pé  da perpendicular traçada de B ao lado CE .

6) observando os quadriláteros inscritíveis : BIER , BIHE , teremos q









Em 23 de agosto de 2013 16:03, douglas.olive...@grupoolimpo.com.brescreveu:

 **

 Olá , alguns alunos do ensino médio da instituição onde trabalho me deram
 alguns problemas do site https://brilliant.org/

 E não consegui achar solução para dois deles, vou escreve-los abaixo e se
 alguém puder me ajudar agradeço.



 PROBLEMA 1: Dada uma função f:R-R tal que f(2x^2 -1)=2(f(x))^2 -1 e f(x)
 é um polinômio de grau 13, sendo assim determine o coeficiente de x^5 de
 f(x).



 PROBLEMA 2: Seja uma circunferência C'  e um ponto externo A , traça-se
 por A duas tangentes a circunferência que a interceptam nos pontos B e C ,
 marca-se no prolongamento de AB no sentido de A para B um ponto D tal que o
 ângulo ADC=25 graus, traça-se por B uma perpendicular ao segmento CD que
 intercepta CD em F . Agora considere um outra circunferência C''
 circunscrita ao triângulo ADC que intercepta a primeira circunferência C'
 no ponto E . Determinar a medida do ângulo DEF.





 Obs: Fiz a segunda figura no geogebra e encontrei 50 graus como resposta ,
 preciso na verdade de uma resolução.



 Att, Douglas Oliveira.


 --
 Esta mensagem foi verificada pelo sistema de antivírus e
 acredita-se estar livre de perigo.


-- 
Esta mensagem foi verificada pelo sistema de antivírus e
 acredita-se estar livre de perigo.



Re: [obm-l] Como que faz??

2013-09-24 Por tôpico Carlos Victor
Olá ,
sem querer dei um enter e a mensagem foi enviada incompleta , também
digitei um ângulo errado no ítem (3) : acompanhe agora :


completando o ítem (6)  e consertando que  no ítem (3) AEB= DEG = z , pois
eu digitei DEB.
teremos que o ângulo BHI = z ; ânguloFIG = 90-x-z  e ângulo EIH = 90-x-z .

7) concluímos então que IJ é bissetriz e altura no triângulo FIH ;
ou seja FJ =JH , daí o triângulo FEH é isósceles .

Temos então que  ângulo FEJ = x ; ou seja

ângulo FED = 2x ... Ufa !!!

Abraços

Carlos Victor



Em 24 de setembro de 2013 19:30, Carlos Victor victorcar...@globo.comescreveu:

 Olá Douglas,
 Acredito ter conseguido uma resolução para o problema 2 de geometria que
 vc postou aqui  .

 Vamos lá  e acompanhe fazendo a figura , ok ?
 vamos provar que na verdade o ângulo DEF é o dobro de ADC.
 Seja  o ângulo  ADC = x e o ângulo CDE = y .

 1) Trace CE e observe que o quadrilátero ACED é inscritível . então AEC =
 x  e  EAC = y .

 2) seja G a intersecção de CD com a circunferência C´ . Trace  EG e
 observe que  o ângulo CGE = x + y . Daí concluímos que o ângulo GED = x .

 3) Não é difícil de mostrar que EB  é bissetriz de AEG . Seja então os
 ângulos AEB= DEB = z .

 4) Trace agora a perpendicular de B  ao segmento ED e seja H o pé desta
 perpendicular. Observe que o quadrilátero BFHD é inscritível , então BHF =
 x .Trace FH e observe que EG é perpendicular  a FH . Seja J a intersecção
 de FH com EG .

 5) Como o triângulo CEG está inscrito na circunferência C´ e observando
 que BF é perpendicular ao lado CD , pelo enunciado ; teremos pela reta de
 SIMSON , que  os pés das  perpendiculares traçadas de B aos lados CG , EG e
 EC  estão alinhados. Sejam então I o pé da perpendicular traçada de ao lado
 EG  e R o pé  da perpendicular traçada de B ao lado CE .

 6) observando os quadriláteros inscritíveis : BIER , BIHE , teremos q









 Em 23 de agosto de 2013 16:03, douglas.olive...@grupoolimpo.com.brescreveu:

 **

 Olá , alguns alunos do ensino médio da instituição onde trabalho me deram
 alguns problemas do site https://brilliant.org/

 E não consegui achar solução para dois deles, vou escreve-los abaixo e se
 alguém puder me ajudar agradeço.



 PROBLEMA 1: Dada uma função f:R-R tal que f(2x^2 -1)=2(f(x))^2 -1 e f(x)
 é um polinômio de grau 13, sendo assim determine o coeficiente de x^5 de
 f(x).



 PROBLEMA 2: Seja uma circunferência C'  e um ponto externo A , traça-se
 por A duas tangentes a circunferência que a interceptam nos pontos B e C ,
 marca-se no prolongamento de AB no sentido de A para B um ponto D tal que o
 ângulo ADC=25 graus, traça-se por B uma perpendicular ao segmento CD que
 intercepta CD em F . Agora considere um outra circunferência C''
 circunscrita ao triângulo ADC que intercepta a primeira circunferência C'
 no ponto E . Determinar a medida do ângulo DEF.





 Obs: Fiz a segunda figura no geogebra e encontrei 50 graus como resposta
 , preciso na verdade de uma resolução.



 Att, Douglas Oliveira.


 --
 Esta mensagem foi verificada pelo sistema de antivírus e
 acredita-se estar livre de perigo.




-- 
Esta mensagem foi verificada pelo sistema de antivírus e
 acredita-se estar livre de perigo.



[obm-l] Re: [obm-l] Álgebra(não tá saindo)

2013-09-15 Por tôpico Carlos Victor
Olá Marcone,

Na hipótese de que quatro vezes maior significa o quádruplo , teremos :

Seja N = y..y6, o número procurado, em que y representa algarismos não
necessariamente iguais . Podemos escrever  N = 10X + 6 .

Logo  4N = 6.(10^n) + X  = 6.( 10^n) + ( N -6)/10 ; ou seja ,

N = 2( 10^(n+1) -1)/13.

Como  10^3 = -1(mod13) , então o menor  N = 2(10^6-1)/13 = 153846 .

Abraços

Carlos  Victor


Em 14 de setembro de 2013 19:15, marcone augusto araújo borges 
marconeborge...@hotmail.com escreveu:

 Encontre o menor inteiro positivo n que possui as seguintes propriedades:
 I. Em sua representação tem o 6 como último dígito
 II.Se o último dígito(6) é apagado  e colocado na frente dos dígitos
 restantes,o número resultante
 é quatro vezes maior que o número original n

 --
 Esta mensagem foi verificada pelo sistema de antivírus e
 acredita-se estar livre de perigo.


-- 
Esta mensagem foi verificada pelo sistema de antivírus e
 acredita-se estar livre de perigo.



Re: [obm-l] trigonometria

2013-08-05 Por tôpico Carlos Victor
Olá  João ,

Esta questão é  de uma olimpíada não brasileira ou de um livro de
olimpíadas ( não lembro qual País), mas encontrar os outros ângulos é um
trabalho árduo e há uma estratégia para a sua solução geométrica . A que
conheço ( em que o mestre Antonio Luis( Gandhi) me mostrou)  é traçar os
simétricos de D e E em relação à  BD e CE , respectivamente, sobre BC .
Faça  uma análise nos triângulos que surgirão , no sentido de que a
bissetriz interna e externa de um triângulo se encontram num ex-incentro e,
aparecerá um ângulo de 120º que é o mentor da solução, ok ? Vale apena
pensar nessa solução ...

Abraços

Carlos Victor


Em 5 de agosto de 2013 11:04, Nehab carlos.ne...@gmail.com escreveu:

  Ora João!

 Nem vem. Você é muito inteligente para odiar Geometria...
 Não acho má ideia você estudar um pouquinho disso...
 Costumo ter sucesso ensinando essa parte maravilhosa da Matemática para
 quem odeia Geometria (hahaha) e gosta de Trigonometria...
 Veja que o ângulo A é imediato... Chamando de I o incentro, segue-se:
 a) No triângulo BIC, ang(BIC) = 180 - (B/2+C/2) = 90 + A/2
 b) No triângulo EID, ang(EID) = 180 - (24 + 18) = 138
 c) Mas ang(BIC) = ang(EID) e daí sai A: 90 + A/2 = 138, ou seja, A = 96

 Tente completar a solução...

 Grande abraço,
 Nehab


 On 04/08/2013 23:37, João Maldonado wrote:

 Fala professor!

 Adorei a resolução, tinha esquecido do 4sen18.cos36 =1   =D
 Na verdade o problema era de geometria, mas como eu sou péssimo em GP,
 sempre resolvo tudo por trigonometria (meu professor fala que eu sou louco)
 O problema era o seguinte:
 Em um triângulo ABC, D e E são os pés das bissetrizes traçadas dos
 vértices B e C respectivamente. CED = 24 graus e BDE = 18 graus, calcule os
 ângulos do triângulo.

 De acordo com o que foi dito os ângulos são 2*36 = 72 graus, 12 graus e 96
 graus

 []'s
 João

  --
 Date: Sat, 3 Aug 2013 23:16:56 -0300
 From: carlos.ne...@gmail.com
 To: obm-l@mat.puc-rio.br
 Subject: Re: [obm-l] trigonometria

 Caramba, João,
 Gostei. Espertinho! Meu raciocínio navegou assim:

 a) 66 = 36 + 30, então 36  é um angulo duplamente interessante pro
 problema.

 b) O que eu sei sobre 36 e companhia? Que o sen18 gosta do cos36 pois
 4sen18.cos36 =1.
 Isso não é exatamente um coelho da cartola, pois essa igualdade é
 clássica  se você estudou os triângulos isósceles que possuem um ângulo de
 36 (trace as diagonais de um pentágono e está tudo lá). Nesses triângulos o
 lado maior é phi vezes o lado menor, ou seja, phi = (raiz(5) + 1)/2 vezes o
 lado menor (uma semelhançazinha). Além disso, esse phi é adorável e é
 manjada razão áurea.
 Dai é fácil você ver nos triângulos isósceles citados (trace as alturas
 deles) que sen18 = 1/2phi e cos36 = phi/2.
 Logo, 4sen18.cos36 = 1...

 c) Assim, achei que seria legal encostar um cos36 no lado direito...

 Então, fica assim:

 tgx = tg 66 - 2sen18/cos66 = [ sen66 - 2sen18] / cos66
 tgx. cos36 = B/C onde
 B = [2sen66cos36 - *4sen18cos36** *] e
 C = 2cos66
 Desenvolvendo B, vem:
 B = sen30 + sen102 - *1* =
 B = sen102 - sen 30 (passagem boba e bonita, né)
 B = 2sen36cos66
 Dai tgx.cos36 = B/C = sen36.
 Logo, x = 36 (se não foi dito que x está entre 0 e 180, então x = 36 +
 k180)

 Abraços
 Nehab

 On 03/08/2013 18:08, João Maldonado wrote:

 tgx = tg66 - 2sen18/cos66
 Como achar x?



 --
 Esta mensagem foi verificada pelo sistema de antivírus e
 acredita-se estar livre de perigo.

 --
 Esta mensagem foi verificada pelo sistema de antivírus e
 acredita-se estar livre de perigo.



 --
 Esta mensagem foi verificada pelo sistema de antivírus e
 acredita-se estar livre de perigo.


-- 
Esta mensagem foi verificada pelo sistema de antivírus e
 acredita-se estar livre de perigo.



Re: [obm-l] trigonometria

2013-08-04 Por tôpico Carlos Victor
Olá grande Mestre Nehab,
Você me ensinou também em 1974 que poderíamos retirar a
igualdade 4sen18.cos36 =1, fazendo :

sen18.cos36 = sen18.cos18.cos36/cos18 = sen36.cos36./2cos18=

sen72/4cos18 = 1/4 .( saudades!!)

Abraços

Carlos Victor


Em 3 de agosto de 2013 23:16, Nehab carlos.ne...@gmail.com escreveu:

  Caramba, João,
 Gostei. Espertinho! Meu raciocínio navegou assim:

 a) 66 = 36 + 30, então 36  é um angulo duplamente interessante pro
 problema.

 b) O que eu sei sobre 36 e companhia? Que o sen18 gosta do cos36 pois
 4sen18.cos36 =1.
 Isso não é exatamente um coelho da cartola, pois essa igualdade é
 clássica  se você estudou os triângulos isósceles que possuem um ângulo de
 36 (trace as diagonais de um pentágono e está tudo lá). Nesses triângulos o
 lado maior é phi vezes o lado menor, ou seja, phi = (raiz(5) + 1)/2 vezes o
 lado menor (uma semelhançazinha). Além disso, esse phi é adorável e é
 manjada razão áurea.
 Dai é fácil você ver nos triângulos isósceles citados (trace as alturas
 deles) que sen18 = 1/2phi e cos36 = phi/2.
 Logo, 4sen18.cos36 = 1...

 c) Assim, achei que seria legal encostar um cos36 no lado direito...

 Então, fica assim:

 tgx = tg 66 - 2sen18/cos66 = [ sen66 - 2sen18] / cos66
 tgx. cos36 = B/C onde
 B = [2sen66cos36 - *4sen18cos36** *] e
 C = 2cos66
 Desenvolvendo B, vem:
 B = sen30 + sen102 - *1* =
 B = sen102 - sen 30 (passagem boba e bonita, né)
 B = 2sen36cos66
 Dai tgx.cos36 = B/C = sen36.
 Logo, x = 36 (se não foi dito que x está entre 0 e 180, então x = 36 +
 k180)

 Abraços
 Nehab


 On 03/08/2013 18:08, João Maldonado wrote:

 tgx = tg66 - 2sen18/cos66
 Como achar x?



 --
 Esta mensagem foi verificada pelo sistema de antivírus e
 acredita-se estar livre de perigo.


-- 
Esta mensagem foi verificada pelo sistema de antivírus e
 acredita-se estar livre de perigo.



Re: [obm-l] trigonometria

2013-08-04 Por tôpico Carlos Victor
Olá Marcone, aproveitando a ideia do meu mestre Nehab, podemos escrever :

acertando a expressão dada chegamos a

sen(66-x) = 2sen18.cosx

tomando 36-x =y ,teremos

sen(30+y) = 2sen18.cos(36-y) = 2sen18[ cos36.cosy + sen36.seny]

sen(30+y) = 2sen18.cos36.cosy + 2sen18.sen36.seny

sen30.cosy+seny.cos30 = 2sen18.cos36.cosy + 2sen18.sen36.seny

usando que o Nehab lembrou , teremos

(1/2)cosy +seny.cos30 =(1/2).cos30 +2sen18.sen36.seny

seny.cos30 = 2sen18.sen36.seny .

Não é difícil de mostrar que 2sen18.sen36 é diferente de cos30 ;

logo devemos ter seny =0 ; ou seja y = k180 ; daí

x= k180 +36 .

Agradecendo ao Nehab ,

Abraços

Carlos Victor


Em 4 de agosto de 2013 13:33, marcone augusto araújo borges 
marconeborge...@hotmail.com escreveu:

 Essa foi muito legal.

 --
 From: ilhadepaqu...@bol.com.br
 To: obm-l@mat.puc-rio.br
 Subject: Re: [obm-l] trigonometria
 Date: Sun, 4 Aug 2013 10:59:12 -0300


 correção
 2014-1974=40 bodas de rubi ou esmeralda, mas mesmo assim merece uma
 comemoração
 Abraço a todos
 Hermann

 - Original Message -
 *From:* Carlos Victor victorcar...@globo.com
 *To:* obm-l@mat.puc-rio.br
 *Sent:* Sunday, August 04, 2013 9:32 AM
 *Subject:* Re: [obm-l] trigonometria

 Olá grande Mestre Nehab,
 Você me ensinou também em 1974 que poderíamos retirar a
 igualdade 4sen18.cos36 =1, fazendo :

 sen18.cos36 = sen18.cos18.cos36/cos18 = sen36.cos36./2cos18=

 sen72/4cos18 = 1/4 .( saudades!!)

 Abraços

 Carlos Victor


 Em 3 de agosto de 2013 23:16, Nehab carlos.ne...@gmail.com escreveu:

  Caramba, João,
 Gostei. Espertinho! Meu raciocínio navegou assim:

 a) 66 = 36 + 30, então 36  é um angulo duplamente interessante pro
 problema.

 b) O que eu sei sobre 36 e companhia? Que o sen18 gosta do cos36 pois
 4sen18.cos36 =1.
 Isso não é exatamente um coelho da cartola, pois essa igualdade é
 clássica  se você estudou os triângulos isósceles que possuem um ângulo de
 36 (trace as diagonais de um pentágono e está tudo lá). Nesses triângulos o
 lado maior é phi vezes o lado menor, ou seja, phi = (raiz(5) + 1)/2 vezes o
 lado menor (uma semelhançazinha). Além disso, esse phi é adorável e é
 manjada razão áurea.
 Dai é fácil você ver nos triângulos isósceles citados (trace as alturas
 deles) que sen18 = 1/2phi e cos36 = phi/2.
 Logo, 4sen18.cos36 = 1...

 c) Assim, achei que seria legal encostar um cos36 no lado direito...

 Então, fica assim:

 tgx = tg 66 - 2sen18/cos66 = [ sen66 - 2sen18] / cos66
 tgx. cos36 = B/C onde
 B = [2sen66cos36 - *4sen18cos36** *] e
 C = 2cos66
 Desenvolvendo B, vem:
 B = sen30 + sen102 - *1* =
 B = sen102 - sen 30 (passagem boba e bonita, né)
 B = 2sen36cos66
 Dai tgx.cos36 = B/C = sen36.
 Logo, x = 36 (se não foi dito que x está entre 0 e 180, então x = 36 +
 k180)

 Abraços
 Nehab


 On 03/08/2013 18:08, João Maldonado wrote:

 tgx = tg66 - 2sen18/cos66
 Como achar x?



 --
 Esta mensagem foi verificada pelo sistema de antivírus e
 acredita-se estar livre de perigo.



 --
 Esta mensagem foi verificada pelo sistema de antivírus e
 acredita-se estar livre de perigo.


 --
 Esta mensagem foi verificada pelo sistema de antivírus e
 acredita-se estar livre de perigo.

 --
 Esta mensagem foi verificada pelo sistema de antivírus e
 acredita-se estar livre de perigo.


-- 
Esta mensagem foi verificada pelo sistema de antivírus e
 acredita-se estar livre de perigo.



Re: [obm-l] trigonometria

2013-08-04 Por tôpico Carlos Victor
Desculpem ,

digitei errado na linha
(1/2)cosy +seny.cos30 =(1/2).cos30 +2sen18.sen36.seny

que na verdade é
(1/2)cosy +seny.cos30 =(1/2).cosy +2sen18.sen36.seny .

Abraços

Carlos  Victor



Em 4 de agosto de 2013 14:09, Carlos Victor victorcar...@globo.comescreveu:

 Olá Marcone, aproveitando a ideia do meu mestre Nehab, podemos escrever :

 acertando a expressão dada chegamos a

 sen(66-x) = 2sen18.cosx

 tomando 36-x =y ,teremos

 sen(30+y) = 2sen18.cos(36-y) = 2sen18[ cos36.cosy + sen36.seny]

 sen(30+y) = 2sen18.cos36.cosy + 2sen18.sen36.seny

 sen30.cosy+seny.cos30 = 2sen18.cos36.cosy + 2sen18.sen36.seny

 usando que o Nehab lembrou , teremos

 (1/2)cosy +seny.cos30 =(1/2).cos30 +2sen18.sen36.seny

 seny.cos30 = 2sen18.sen36.seny .

 Não é difícil de mostrar que 2sen18.sen36 é diferente de cos30 ;

 logo devemos ter seny =0 ; ou seja y = k180 ; daí

 x= k180 +36 .

 Agradecendo ao Nehab ,

 Abraços

 Carlos Victor


 Em 4 de agosto de 2013 13:33, marcone augusto araújo borges 
 marconeborge...@hotmail.com escreveu:

  Essa foi muito legal.

 --
 From: ilhadepaqu...@bol.com.br
 To: obm-l@mat.puc-rio.br
 Subject: Re: [obm-l] trigonometria
 Date: Sun, 4 Aug 2013 10:59:12 -0300


 correção
 2014-1974=40 bodas de rubi ou esmeralda, mas mesmo assim merece uma
 comemoração
 Abraço a todos
 Hermann

 - Original Message -
 *From:* Carlos Victor victorcar...@globo.com
 *To:* obm-l@mat.puc-rio.br
 *Sent:* Sunday, August 04, 2013 9:32 AM
 *Subject:* Re: [obm-l] trigonometria

 Olá grande Mestre Nehab,
 Você me ensinou também em 1974 que poderíamos retirar a
 igualdade 4sen18.cos36 =1, fazendo :

 sen18.cos36 = sen18.cos18.cos36/cos18 = sen36.cos36./2cos18=

 sen72/4cos18 = 1/4 .( saudades!!)

 Abraços

 Carlos Victor


 Em 3 de agosto de 2013 23:16, Nehab carlos.ne...@gmail.com escreveu:

  Caramba, João,
 Gostei. Espertinho! Meu raciocínio navegou assim:

 a) 66 = 36 + 30, então 36  é um angulo duplamente interessante pro
 problema.

 b) O que eu sei sobre 36 e companhia? Que o sen18 gosta do cos36 pois
 4sen18.cos36 =1.
 Isso não é exatamente um coelho da cartola, pois essa igualdade é
 clássica  se você estudou os triângulos isósceles que possuem um ângulo de
 36 (trace as diagonais de um pentágono e está tudo lá). Nesses triângulos o
 lado maior é phi vezes o lado menor, ou seja, phi = (raiz(5) + 1)/2 vezes o
 lado menor (uma semelhançazinha). Além disso, esse phi é adorável e é
 manjada razão áurea.
 Dai é fácil você ver nos triângulos isósceles citados (trace as alturas
 deles) que sen18 = 1/2phi e cos36 = phi/2.
 Logo, 4sen18.cos36 = 1...

 c) Assim, achei que seria legal encostar um cos36 no lado direito...

 Então, fica assim:

 tgx = tg 66 - 2sen18/cos66 = [ sen66 - 2sen18] / cos66
 tgx. cos36 = B/C onde
 B = [2sen66cos36 - *4sen18cos36** *] e
 C = 2cos66
 Desenvolvendo B, vem:
 B = sen30 + sen102 - *1* =
 B = sen102 - sen 30 (passagem boba e bonita, né)
 B = 2sen36cos66
 Dai tgx.cos36 = B/C = sen36.
 Logo, x = 36 (se não foi dito que x está entre 0 e 180, então x = 36 +
 k180)

 Abraços
 Nehab


 On 03/08/2013 18:08, João Maldonado wrote:

 tgx = tg66 - 2sen18/cos66
 Como achar x?



 --
 Esta mensagem foi verificada pelo sistema de antivírus e
 acredita-se estar livre de perigo.



 --
 Esta mensagem foi verificada pelo sistema de antivírus e
 acredita-se estar livre de perigo.


 --
 Esta mensagem foi verificada pelo sistema de antivírus e
 acredita-se estar livre de perigo.

 --
 Esta mensagem foi verificada pelo sistema de antivírus e
 acredita-se estar livre de perigo.




-- 
Esta mensagem foi verificada pelo sistema de antivírus e
 acredita-se estar livre de perigo.



[obm-l] Re: [obm-l] Re: [obm-l] RE: [obm-l] Re: [obm-l] Soluções inteiras da equação x/y = x - y

2013-06-18 Por tôpico Carlos Victor
Olá ,
É interessante também  observar que nesses tipos de problemas , já que y=0
e  y =1 não são soluções, podemos escrever :

x = y^2/(y-1) = y+1 +1/(y-1) ; ou seja (y-1) deve ser -1 ou +1 . Daí y = 2
e x = 4 .

Abraços

Carlos Victor



Em 18 de junho de 2013 19:43, Marcelo Salhab Brogliato
msbro...@gmail.comescreveu:

 É verdade! Nesse caso, chega-se a mesma conclusão, mas em outros problemas
 esse erro pode esconder alguma possível solução.

 Obrigado! :)

 Abraços,
 Salhab


 2013/6/18 Paulo Argolo pauloarg...@outlook.com

 Caro Salhab,

 Na verdade:  k|y e y|k = |k| = |y|
 De qualquer forma, chega-se a mesma conclusão.

 Um abraço do Paulo Argolo!
 ___



 Date: Tue, 18 Jun 2013 15:14:58 -0300
 Subject: [obm-l] Re: [obm-l] Soluções inteiras da equação x/y = x - y
 From: msbro...@gmail.com
 To: obm-l@mat.puc-rio.br

 Olá, Ennius, tudo bem?

 Se as soluções são inteiras, então temos que y|x, logo: x = ky. Assim:
 ky/y = ky - y

 k = ky - y
 k + y = ky

 Então: k|y e y|k = y = k.

 y + y = y*y = y(y-2) = 0 = y = 0 ou y = 2. Mas y não pode ser 0, pois a
 equação original é x/y = x - y.
 Assim: y = 2, k = 2 e x = ky = 4.

 Abraços,
 Salhab


 2013/6/18 ennius enn...@bol.com.br
 Colegas da Lista,

 Como mostrar que a equação x/y = x - y não admite soluções inteiras,
  além de x = 4 e y = 2?
 --

 --
 Esta mensagem foi verificada pelo sistema de antivírus e
  acredita-se estar livre de perigo.

 =
 Instruções para entrar na lista, sair da lista e usar a lista em
 http://www.mat.puc-rio.br/~obmlistas/obm-l.html
 =


 --
 Esta mensagem foi verificada pelo sistema de antivírus e
 acredita-se estar livre de perigo.
 --
 Esta mensagem foi verificada pelo sistema de antivírus e
  acredita-se estar livre de perigo.


 =
 Instruções para entrar na lista, sair da lista e usar a lista em
 http://www.mat.puc-rio.br/~obmlistas/obm-l.html
 =



 --
 Esta mensagem foi verificada pelo sistema de antivírus e
 acredita-se estar livre de perigo.


-- 
Esta mensagem foi verificada pelo sistema de antivírus e
 acredita-se estar livre de perigo.



[obm-l] Re: [obm-l] Duas perguntas(teoria dos números)

2013-05-27 Por tôpico Carlos Victor
Olá ,
Observando que m+48 = 2^k e m-48 = 2^(n-k) ,
teremos  3 = 2^(k-5) - 2^(n-k-5) ; ou seja k - 5 =2 e n-k-5 = 0 .

Então n =12 . Está Ok isso ?

Carlos Victor




Em 27 de maio de 2013 14:16, marcone augusto araújo borges 
marconeborge...@hotmail.com escreveu:

 1) Gostaria de saber se a soma de duas ou mais potencias de base 2
 distintas pode ser uma potencia de base 2.

 Acredito que não e escrevendo esses números na base 2 talvez se possa
 mostrar isso.

 2) Desconfio que   2304 + 2^n é um quadrado perfeito para um único valor
 de n.

 Eu fiz 2^n = (m + 48)(m - 48)
 m + 48 e m - 48 devem ser potencias de base 2
 As únicas potencias de base 2 cuja diferença é 96 são 128 e 32
 Dai o único valor de n seria 12
 Um esclarecimento seria muito bem vindo

 Desde já agradeço





Re: [obm-l] Problema de Geometria

2013-04-28 Por tôpico Carlos Victor
Olá Raphael,
Pense no seguinte :

1) Trace OC
2) Trace BD
3) Conclua que BD é o dobro de OC.
4) Denomine EF = x
5) Faça  a semelhança de OCF com BFD e determine x , ok ?

Abraços

Carlos  Victor


Em 28 de abril de 2013 18:19, Raphael Feijao
raphaelfei...@hotmail.comescreveu:

 O segmento AB é o diametro de uma circunferencia de centro O. Toma-se um
 ponto C desse círculo e prolonga-se o segmento AC de um segmento CD igual a
 AC. O segmento OD corta a circunferencia em E e corta o segmento BC em F.
 Se AB=a e OD=b. Calcule EF.



[obm-l] Re: [obm-l] Potência encardida

2013-04-02 Por tôpico Carlos Victor
Olá Vanderlei ,
O que vc pode perceber que  na sequência 2^2, 2^22,2^42,..., todos terminam
em 04 . 2^222 está nesta sequência , ok ?

Abraços

Carlos Victor

Em 2 de abril de 2013 13:01, Vanderlei * vanderma...@gmail.com escreveu:

 *Bom dia, pessoal! Gostaria de uma ajuda na seguinte questão, a qual eu
 só consegui com binômio de Newton e alguma força bruta.*
 **
 *Quais são os dois últimos algarismos do resultado de 2^222?*
 **
 *A resposta é 04.*
 **
 *Obrigado!*
 **
 *Vanderlei*

 --
 Esta mensagem foi verificada pelo sistema de antivírus e
 acredita-se estar livre de perigo.

-- 
Esta mensagem foi verificada pelo sistema de antivírus e
 acredita-se estar livre de perigo.



Re: [obm-l] Desigualdades

2013-03-23 Por tôpico Carlos Victor
Ok,  Meu Grande Mestre Nehab,

Um Saudoso Abraço

Carlos

Victor

Em 20 de março de 2013 23:21, Nehab carlos.ne...@gmail.com escreveu:

  Oi, querido amigo,

 Apenas uma observação:
 Ficou provado que 96 majora a soma, mas ainda temos que explicitar x, y e
 z com xyz = 32 que faz a soma ser IGUAL a 96.
 Em sua prova a igualdade a 96 valeria se houvesse x, y e z com 4xy = z^2
 (e naturalmente xyz = 32).
 De fato isto ocorre qdo z = 4 e dai, x =4 e y = 2.

 Um grande abraço,
 Saudades
 Nehab


 On 20/03/2013 08:51, Carlos Victor wrote:

 Olá ,
 acredito que dê  só por médias :
 4xy + (x^2 + 4y^2) + 2z^2 = 4xy + 4xy + 2z^2 = 3.raiz cúbica de (
 32(xyz)^2) =3.32 = 96.

 Carlos Victor


 Em 19 de março de 2013 20:41, Bernardo Freitas Paulo da Costa 
 bernardo...@gmail.com escreveu:

 2013/3/19 Carlos Yuzo Shine cysh...@yahoo.com:
  Só para evitar derivadas (especialmente de mais de uma variável, em que
 há vários detalhes), aí vão soluções:
 
  1) Pela desigualdade de médias, a expressão é igual a 4xy + (x^2 +
 4y^2) + 2z^2 = 4xy + 4xy + 2z^2 = 8xy + 2z^2 = 4xyz = 4*32 = 128. A
 igualdade ocorre quando x = 2y e 4xy = z^2, ou seja, x = 2^(11/6), y =
 2^(5/6) e z = 2^(7/3).

  Oi Shine,

 eu não entendi a passagem 8xy + 2z^2 = 4xyz. Não pode ser só
 desigualdade das médias, porque essa é homogênea, e todos os termos da
 esquerda são de ordem dois. Acho que faltou uma dica para o seu caro
 leitor.

 Pensando um pouco mais, eu resolveria com multiplicadores de Lagrange
 (e portanto com derivadas). Mas se fosse antes de aprender Lagrange,
 eu teria feito assim:

 Note que se z é fixo, temos que minimizar (x + 2y)^2, com xy =
 constante. (Aplicando a famosa técnica escolha produtos notáveis que
 vão te ajudar.) Pela MA = MG, obtemos x = 2y (como todo mundo
 obteve...).

 xy = 32/z, x = 2y = 2y^2 = 32/z = y^2 = 16/z, x^2 = 4*16/z e
 portanto x^2 + 4xy + 4y^2 = 4*16/z + 4*32/z + 4*16/z = 4*32*2/z.

 Queremos minimizar 4*32*2/z + 2z^2. Pela desigualdade das médias com 3
 termos: 4*32/z + 4*32/z + 2z^2 = 3 * (4*32 * 4*32 * 2)^1/3 = 3 *
 (2^(2+5+2+5+1))^1/3 = 3 * 2^5 = 3 * 32 = 96. A igualdade ocorre para

 4*32/z = 2z^2 = 64 = z^3, ou seja z = 4, y = 4/raiz(z) = 4/2 = 2, x = 4.

 Verificando: x^2 = 4^2 = 16
 4xy = 4*2*4 = 32
 4*y^2 = 4*2^2 = 16
 2z^2 = 2*4^2 = 32
 Somando = 96.

 --
 Bernardo Freitas Paulo da Costa

 =
 Instruções para entrar na lista, sair da lista e usar a lista em
 http://www.mat.puc-rio.br/~obmlistas/obm-l.html
 =






Re: [obm-l] Desigualdades

2013-03-20 Por tôpico Carlos Victor
Olá ,
acredito que dê  só por médias :
4xy + (x^2 + 4y^2) + 2z^2 = 4xy + 4xy + 2z^2 = 3.raiz cúbica de (
32(xyz)^2) =3.32 = 96.

Carlos Victor


Em 19 de março de 2013 20:41, Bernardo Freitas Paulo da Costa 
bernardo...@gmail.com escreveu:

 2013/3/19 Carlos Yuzo Shine cysh...@yahoo.com:
  Só para evitar derivadas (especialmente de mais de uma variável, em que
 há vários detalhes), aí vão soluções:
 
  1) Pela desigualdade de médias, a expressão é igual a 4xy + (x^2 + 4y^2)
 + 2z^2 = 4xy + 4xy + 2z^2 = 8xy + 2z^2 = 4xyz = 4*32 = 128. A igualdade
 ocorre quando x = 2y e 4xy = z^2, ou seja, x = 2^(11/6), y = 2^(5/6) e z =
 2^(7/3).

 Oi Shine,

 eu não entendi a passagem 8xy + 2z^2 = 4xyz. Não pode ser só
 desigualdade das médias, porque essa é homogênea, e todos os termos da
 esquerda são de ordem dois. Acho que faltou uma dica para o seu caro
 leitor.

 Pensando um pouco mais, eu resolveria com multiplicadores de Lagrange
 (e portanto com derivadas). Mas se fosse antes de aprender Lagrange,
 eu teria feito assim:

 Note que se z é fixo, temos que minimizar (x + 2y)^2, com xy =
 constante. (Aplicando a famosa técnica escolha produtos notáveis que
 vão te ajudar.) Pela MA = MG, obtemos x = 2y (como todo mundo
 obteve...).

 xy = 32/z, x = 2y = 2y^2 = 32/z = y^2 = 16/z, x^2 = 4*16/z e
 portanto x^2 + 4xy + 4y^2 = 4*16/z + 4*32/z + 4*16/z = 4*32*2/z.

 Queremos minimizar 4*32*2/z + 2z^2. Pela desigualdade das médias com 3
 termos: 4*32/z + 4*32/z + 2z^2 = 3 * (4*32 * 4*32 * 2)^1/3 = 3 *
 (2^(2+5+2+5+1))^1/3 = 3 * 2^5 = 3 * 32 = 96. A igualdade ocorre para

 4*32/z = 2z^2 = 64 = z^3, ou seja z = 4, y = 4/raiz(z) = 4/2 = 2, x = 4.

 Verificando: x^2 = 4^2 = 16
 4xy = 4*2*4 = 32
 4*y^2 = 4*2^2 = 16
 2z^2 = 2*4^2 = 32
 Somando = 96.

 --
 Bernardo Freitas Paulo da Costa

 =
 Instruções para entrar na lista, sair da lista e usar a lista em
 http://www.mat.puc-rio.br/~obmlistas/obm-l.html
 =



[obm-l] Re: [obm-l] Geometria Plana - Triângulo

2012-08-28 Por tôpico Carlos Victor
Olá  Arkon ,
Uma solução é :

Seja O o ortocentro de ABC . Observe que o triângulo AOC é semelhante ao
triângulo OEH  , pois o quadrilátero ACHE é inscritível . Seja x  = EH ,
então 7/x = AO/EO e como OE = OA.cosB . Usando a lei dos cosenos  encontre
cosB = 1/5 e daí x =7/5 , ok ? .Acredito que pensar no círculo dos nove
pontos  pode também resolver .

Confira as contas .

Abraços

Carlos  Victor

Em 28 de agosto de 2012 19:31, arkon ar...@bol.com.br escreveu:

 Pessoal, qual o bizu?
 Â
 Em um triângulo ABC, traçam-se as alturas AH e CE. Se AB=5m, BC=6m e
 AC=7m, calcule EH.
 Â
 (A) 7/5 m (B) 9/5 m (C) 10/7 m (D) 10/3 m (E) 2
 =
 Instruções para entrar na lista, sair da lista e usar a lista em
 http://www.mat.puc-rio.br/~obmlistas/obm-l.html=


Re: [obm-l] Divisibilidade

2012-05-20 Por tôpico Carlos Victor
Olá Thiago ,

Pense assim :

43x+75y = 38x +76y  + 5x -y

Basta então mostrar que  5x-y é múltiplo de 19 .

5x-y = 5(5x-y) - 2(3x+7y) = 19x - 19y . Como  3x+7y =19k , temos que 43x+
75y também é .

Abraços

Carlos  Victor

Em 11 de maio de 2012 08:25, Thiago Bersch thiago_t...@hotmail.comescreveu:

  Mostre que se [image: 19|3x+7y] então [image: 19|43x+75y]



[obm-l] Re: [obm-l] Congruência modulo n

2011-12-16 Por tôpico Carlos Victor
Olá  Kleber ,

Usando o teorema de Euler temos que  12^20 é congruo a 1 mod (25). Elevando
a 657 , temos que  12^13140 é congruo 1 mod(25).Logo , basta ver a divisão
de 12^5 por 25 , ok ?.

Teorema de Euler :Sejam a,m naturais com m  1 e mdc(a,m) =1. Então  a^(fi
de m) é congruo a 1 modm .

Abraços

Carlos  Victor

Em 16 de dezembro de 2011 13:49, Kleber Bastos klebe...@gmail.comescreveu:


 Queria saber qual o método para calcular:
 Dado que 12^13145(mod 25), calcular o resto da divisão de 12^13145 por 25.

 Desde já agradeço a ajuda.
 Abraços, Kleber.





[obm-l] Re: [obm-l] Exercício proposto(Eureka!)

2011-07-28 Por tôpico Carlos Victor
Olá Marcone ,
Sabendo  que  : cos(pi/7) - cos(2pi/7) + cos(3pi/7) = 1/2 , use as
expressões de  cos3x  e de  cos2x  em função de cosx , com  x = pi/7 ,  ok ?

Abraços

Carlos  Victor

Em 28 de julho de 2011 18:24, marcone augusto araújo borges 
marconeborge...@hotmail.com escreveu:

  Prove que x = 2cos(pi/7) satisfaz a equação x^3 + x^2 -2x + 1 = 0.

 Há um exercício resolvido na revista,mostrando que pi/7 é raíz da equação
 8x^3 - 4x^2 -4x + 1 = 0
 Mas não estou conseguindo e peço ajuda
 Agradeço desde já.






Re: [obm-l] Geometria OBM

2011-07-24 Por tôpico Carlos Victor
Olá João ,

Vamos inicialmente a uma solução trigonométrica :

Seja z o ângulo pedido .Sejam também AB =a ; AO2 = x  e AO1= y.Então teremos
:

Triang  *BO1O2 : y/sen(160-z) = x/sen*z

Triang ABO2 : x/sen50 = a/sen80

Triang BO1A : y/sen80 = a/sen70

Logo :sen(20+z) = 4cos10.sen20.senz  ; ou  sen(20+z) -senz = 2senz.sen10

Donde  cos(z+10) = senz ; ou seja  z = 40 ° .

Tentarei uma solução geométrica . ok ?

Abraços

Carlos  Victor





2011/7/24 João Maldonado joao_maldona...@hotmail.com

  Inglaterra -- 1970

 No triângulo ABC, AB = AC e A=80°,  dado O1 em AC tal que O1BC =  20° e O2
 em  BC tal que   CAO2 = 30°,  calcule   BO1O2

 Obrigado
 João



Re: [obm-l] Geometria OBM

2011-07-24 Por tôpico Carlos Victor
Olá João ,

Houve um erro na digitação : onde está  AO1 =y  lê-se  BO1 =y . ok ? .
Desculpe o erro .

Abraços

Carlos  Victor

Em 24 de julho de 2011 19:11, Carlos Victor victorcar...@globo.comescreveu:

 Olá João ,

 Vamos inicialmente a uma solução trigonométrica :

 Seja z o ângulo pedido .Sejam também AB =a ; AO2 = x  e AO1= y.Então
 teremos :

 Triang  *BO1O2 : y/sen(160-z) = x/sen*z

 Triang ABO2 : x/sen50 = a/sen80

 Triang BO1A : y/sen80 = a/sen70

 Logo :sen(20+z) = 4cos10.sen20.senz  ; ou  sen(20+z) -senz = 2senz.sen10

 Donde  cos(z+10) = senz ; ou seja  z = 40 ° .

 Tentarei uma solução geométrica . ok ?

 Abraços

 Carlos  Victor






 2011/7/24 João Maldonado joao_maldona...@hotmail.com

  Inglaterra -- 1970

 No triângulo ABC, AB = AC e A=80°,  dado O1 em AC tal que O1BC =  20° e O2
 em  BC tal que   CAO2 = 30°,  calcule   BO1O2

 Obrigado
 João





Re: [obm-l] Re: [obm-l] Geometria OBM

2011-07-24 Por tôpico Carlos Victor
Linda Solução
 do Julio.

Carlos Victor

2011/7/24 Julio César Saldaña saldana...@pucp.edu.pe



 Uma solução geométrica:

 Sabemos que O2A=O2B.
 Prolongue ou estique BO1, (desculpem o protunhol) até um ponto P tal que
 O2P=O2B
 (=O2A). Calculemos uns ángulos: BPO2=20, PO2C=40, AO2P=60, então o
 triángulo
 O2AP é equilátero, ouseja PA=AO2, PAC=30 e a reta AO1 será mediatriz de
 PO2.
 Então PO1=O1O2, então O1O2P=20, finalmente  BO1O2=20+20=40



 Julio Saldaña


 -- Mensaje original ---
 De : obm-l@mat.puc-rio.br
 Para : obm-l@mat.puc-rio.br
 Fecha : Sun, 24 Jul 2011 19:18:18 -0300
 Asunto : Re: [obm-l] Geometria OBM
 Olá João ,
 
 Houve um erro na digitação : onde está  AO1 =y  lê-se  BO1 =y . ok ? .
 Desculpe o erro .
 
 Abraços
 
 Carlos  Victor
 
 Em 24 de julho de 2011 19:11, Carlos Victor victorcar...@globo.com
 escreveu:
 
  Olá João ,
 
  Vamos inicialmente a uma solução trigonométrica :
 
  Seja z o ângulo pedido .Sejam também AB =a ; AO2 = x  e AO1= y.Então
  teremos :
 
  Triang  *BO1O2 : y/sen(160-z) = x/sen*z
 
  Triang ABO2 : x/sen50 = a/sen80
 
  Triang BO1A : y/sen80 = a/sen70
 
  Logo :sen(20+z) = 4cos10.sen20.senz  ; ou  sen(20+z) -senz = 2senz.sen10
 
  Donde  cos(z+10) = senz ; ou seja  z = 40 ° .
 
  Tentarei uma solução geométrica . ok ?
 
  Abraços
 
  Carlos  Victor
 
 
 
 
 
 
  2011/7/24 João Maldonado joao_maldona...@hotmail.com
 
   Inglaterra -- 1970
 
  No triângulo ABC, AB = AC e A=80°,  dado O1 em AC tal que O1BC =  20° e
 O2
  em  BC tal que   CAO2 = 30°,  calcule   BO1O2
 
  Obrigado
  João
 
 
 
 

 __
 Si desea recibir, semanalmente, el Boletín Electrónico de la PUCP, ingrese
 a:
 http://www.pucp.edu.pe/puntoedu/suscribete/

 =
 Instruções para entrar na lista, sair da lista e usar a lista em
 http://www.mat.puc-rio.br/~obmlistas/obm-l.html
 =



Re: [obm-l] Fwd: Identidade de Euler

2011-04-28 Por tôpico Carlos Victor
Oi  Mestre  Nehab ,
Gostei da sugestão e mais ainda  das n pessoas que moram em Nilópolis (
minha  terrinha).

Abraços

Carlos  Victor

Em 28 de abril de 2011 17:21, Carlos Nehab ne...@infolink.com.br escreveu:

 Oi, Fábio,

 Não resisti:

 Resolva os seguinte problema de duas maneiras (uma técnica básica e útil
 para resolver identidades deste tipo).
 De quantas maneira posso formar comissões de p pessoas, a partir de um
 total de m + n pessoas, sendo m o total de pessoas que moram no Maracanã e n
 as pessoas que moram em Nilópolis?

 Abraços,
 Nehab

 Em 28/4/2011 13:24, fabio henrique teixeira de souza escreveu:

 -- Mensagem encaminhada --
 De: fabio henrique teixeira de souzafabiodja...@ig.com.br
 Data: 28 de abril de 2011 08:52
 Assunto: Identidade de Euler
 Para: obm-l@mat.puc-rio.br


 Pessoal, estou batendo cabeça e não consigo demonstrar que
 C(m,0).C(n,p) + C(m,1).C(n,p-1) + C(m,2).C(n,p-2) + ... + C(m,p).C(n,0) =
 C(m+n,p)

 Alguém pode me dar uma dica?


 =
 Instruções para entrar na lista, sair da lista e usar a lista em
 http://www.mat.puc-rio.br/~obmlistas/obm-l.html
 =



Re: [obm-l] distancia no trapezio

2011-01-09 Por tôpico Carlos Victor
Olá  Luis,

Trace de D uma paralela ao lado BC que encontra AB por exemplo em E. Observe
que BE = 4 e o triângulo EDA é isósceles  e, donde EA = 6. Logo AB = 10 , ok
? . Verifique se esta ideia está correta .

Abraços

Carlos Victor

Em 9 de janeiro de 2011 13:56, Luís Lopes qed_te...@hotmail.com escreveu:

  Sauda¸c~oes,

 Pediram-me para resolver o seguinte problema.

 No trapezio ABCD, CD=4 e DA=6. E tambem B=\alpha e D=2\alpha.
 Calcular AB.

 O desenho do trapezio eh

   C o---o D



 B o---o A


 Abra¸cos,
 Luis




RE: [obm-l] Ajuda URGENTE

2007-01-22 Por tôpico Carlos Victor


Olá  ,
Trace  a  altura  AH  relativa  ao  lado  BC . Escolha  um 
ponto  Q  sobre  AH , tal  que  o  triângulo BCQ  seja  equilátero . 
Agora  , verifique  a  congruência  dos  triângulos  ABQ  e 
BPA   e,  daí  conclua  que  o  o ângulo  BQA = 150°  e consequentemente  o 
ângulo  pedido   será  30°, ok ?


[]´s  Carlos  Victor







At 16:56 22/1/2007, Filipe de Carvalho Hasché wrote:

Em um triângulo ABC, tem-se que os ângulos ABC = ACB = 80º. Se P é um ponto
sobre o lado AB tal que AP = BC, a medida do ângulo BPC é igual a:


=

Eu emperrei tb, heheheh
Mas fiz no Cabri e deu 30°.
E nem assim consegui desvendar..

Abraços,
FC.

_
MSN Busca: fácil, rápido, direto ao ponto.  http://search.msn.com.br

=
Instruções para entrar na lista, sair da lista e usar a lista em
http://www.mat.puc-rio.br/~nicolau/olimp/obm-l.html
=




=
Instruções para entrar na lista, sair da lista e usar a lista em
http://www.mat.puc-rio.br/~nicolau/olimp/obm-l.html
=


Re: [obm-l] Inducao

2007-01-17 Por tôpico Carlos Victor


Olá  Luís ,
Divida  inicialmente  o quadrado  original em  16  quadradinhos  , 
apague  os  9  quadradinhos  do  canto  superior  esquerdo 
(por  exemplo)  , ficará  um  quadrado  maior  , junto  com 
os  outros   7   quadradinhos  que  sobraram , ok  ?


Abraços   e  satisfação em falar  com você

[]´s  Carlos  Victor






At 12:17 17/1/2007, Luís Lopes wrote:

Sauda,c~oes,

Oi Carlos Victor,

Como obter 8 quadrados?

Seguindo suas idéias dividi o quadrado inicial em
9 e 16 quadrados iguais. Com os 9 quadrados
gera-se a seqüência 6,9,12, E com os 16,
a seqüência 4,7,10,13,16,19...

A 8,11,14,... não consegui. Talvez se eu soubesse
resolver 3k+8=n^2 ajudasse.

[]'s
Luís


From: Carlos  Victor [EMAIL PROTECTED]
Reply-To: obm-l@mat.puc-rio.br
To: obm-l@mat.puc-rio.br, obm-l@mat.puc-rio.br
Subject: Re: [obm-l] Inducao
Date: Tue, 16 Jan 2007 19:58:49 -0200


Olá  Klaus, para  o segundo :

Observe  que  quando dividimos  um quadrado  em 4 partes , na verdade
acrescentamos  3 quadradinhos  ao  quadrado original . Pensando  desta
forma basta você  conseguir  dividir  um  quadrado   em 6 , 7 e 8  outros
quadradinhos, pois  a partir  desses  usa  o procedimento inicial . 
Com  um pouco  de paciência  verifica-se  que   dividir  um 
quadrado  em  6 , 7  e

8  outros  quadradinhos   não é  difícil  e ,  consequentemente  teremos
as seguintes   sequências  :

1) 6 ,9 , 12 , ...

2) 7 , 10 , 13 , ...

3) 8 , 11 , 14 , ...

Unindo  as sequências  temos  os  naturais   a partir  de  6 , ok ?


[]´s  Carlos  Victor




At 18:27 16/1/2007, Klaus Ferraz wrote:
1)Prove que todo inteiro positivo pode ser escrito como potencias de 2 
com expoentes distintos

2)Prove que um quadrado pode ser dividido em n quadrados para n=6.
3)Prove que [1.3.5..(2n-1)]/[2.4.6.8...2n]=1/sqrt(2n+1)

Grato.


_
MSN Busca: fácil, rápido, direto ao ponto.  http://search.msn.com.br

=
Instruções para entrar na lista, sair da lista e usar a lista em
http://www.mat.puc-rio.br/~nicolau/olimp/obm-l.html
=




=
Instruções para entrar na lista, sair da lista e usar a lista em
http://www.mat.puc-rio.br/~nicolau/olimp/obm-l.html
=


Re: Res: [obm-l] Inducao

2007-01-17 Por tôpico Carlos Victor


Olá  Klaus ,

Observe o seguinte : inicialmente eu estou vendo um quadrado , quando o 
dividimos  em quatro partes , ficam  4 partes  menores , ok ? ; ou seja , 
eu tinha  um quadrado  e  agora  eu tenho 4 quadradinhos . Daí 
acrescentei  3  , ok ?


[]´s   Carlos  Victor




At 16:55 17/1/2007, Klaus Ferraz wrote:

Ola Carlos
 Nao entendi. Porque quando dividimos um quadrado em 
4 partes estamos acrescentando 3 quadradinhos ao quadrado original


-,




=
Instruções para entrar na lista, sair da lista e usar a lista em
http://www.mat.puc-rio.br/~nicolau/olimp/obm-l.html
=


Re: [obm-l] Inducao

2007-01-16 Por tôpico Carlos Victor


Olá  Klaus, para  o segundo :

Observe  que  quando dividimos  um quadrado  em 4 partes , na 
verdade  acrescentamos  3 quadradinhos  ao  quadrado original . 
Pensando  desta  forma basta você  conseguir  dividir  um  quadrado   em 6 
, 7 e 8  outros  quadradinhos, pois  a partir  desses  usa  o procedimento 
inicial . Com  um pouco  de paciência  verifica-se  que   dividir  um 
quadrado  em  6 , 7  e  8  outros  quadradinhos   não é  difícil  e 
,  consequentemente  teremos  as seguintes   sequências  :


1) 6 ,9 , 12 , ...

2) 7 , 10 , 13 , ...

3) 8 , 11 , 14 , ...

Unindo  as sequências  temos  os  naturais   a partir  de  6 , ok ?


[]´s  Carlos  Victor








At 18:27 16/1/2007, Klaus Ferraz wrote:
1)Prove que todo inteiro positivo pode ser escrito como potencias de 2 com 
expoentes distintos

2)Prove que um quadrado pode ser dividido em n quadrados para n=6.
3)Prove que [1.3.5..(2n-1)]/[2.4.6.8...2n]=1/sqrt(2n+1)

Grato.


__
Fale com seus amigos de graça com o novo Yahoo! Messenger
http://br.messenger.yahoo.com/




=
Instruções para entrar na lista, sair da lista e usar a lista em
http://www.mat.puc-rio.br/~nicolau/olimp/obm-l.html
=


Re: [obm-l] PG

2006-12-29 Por tôpico Carlos Victor


Olá   Marcus ,

Faça o  seguinte : suponha  que  as ordens de  1, 2  e 5  sejam 
respectivamente  m , p  e  n . Use  a expressão  do termo geral  com 
razão  igual  a q  e  conclua  que  :


5^(p-m) = 2^(n-m)  e já  que   m,n e p são  naturais  , teremos p=m=n 
.  Conclusão : 1,2e5  não podem  pertencer  a uma  mesma  PG , ok ? 
confira  as contas.


[]´s  Carlos  Victor




At 10:06 29/12/2006, Marcus Aurélio wrote:

Podem os números 1, 2 e 5 pertencer à mesma PG



=
Instruções para entrar na lista, sair da lista e usar a lista em
http://www.mat.puc-rio.br/~nicolau/olimp/obm-l.html
=



=
Instruções para entrar na lista, sair da lista e usar a lista em
http://www.mat.puc-rio.br/~nicolau/olimp/obm-l.html
=


Re: [obm-l] probleminha da en

2006-12-11 Por tôpico Carlos Victor

Olá  Arkon,

Como  dizia  o nosso mestre MORGADO ,  um  truque   para  este tipo  de 
problema é :


Como  são  quatro  conjuntos , o que  ultrapassar  a  300%  será  a 
quantidade   da interseção  dos  conjuntos . Se  tivermos  n  conjuntos , o 
que  ultrapasar  a (n-1)x100%   será  o mínimo  da interseção , ( tente 
provar) ,ok ?


[]´s   Carlos  Victor




At 16:58 11/12/2006, arkon wrote:
Gostaria que alguém da lista me enviasse a resolução de mais uma questão 
da en, por favor:


grato.

Se 70% da população gostam de samba, 75% de choro, 80% de bolero e 85% de 
rock, quantos por cento da população, no mínimo, gostam de samba, choro, 
bolero e rock?


a) 5%.
b) 10%.
c) 20%.
d) 45%.
e) 70%.

Obs.: A alternativa correta é a letra b.



=
Instruções para entrar na lista, sair da lista e usar a lista em
http://www.mat.puc-rio.br/~nicolau/olimp/obm-l.html
=


Re: [obm-l] ajuda em tres questoes

2006-12-06 Por tôpico Carlos Victor


Olá  Fábio ,

1) Para  a primeira  questão  faça  o seguinte : agrupar  onde tiver  o 
parâmetro   a  e  depois  anular  o fator  que estiver  multiplicando  o 
parâmetro . Neste  momento  você irá  encontrar  uma equação do seg 
grau  em  x  e  consequentemente  encontrará  dois  pontos  no plano  e 
daí  achar  a distância  entre eles, ok ?


2)  Para  o segundo : trace  a bissetriz  do ângulo  dobrado  e  utilize  o 
teorema  da bissetriz  interna junto  com uma semelhança  que irá  aparecer 
, ok ?


3) Para o terceiro : faça  duas  semelhanças   e utilize  o fato  de que  o 
raio do circulo  inscrito é dado por  p-a , onde p é  o semi-perímetro  e 
a é a hipotenusa , ok ?


[]´s   Carlos  Victor








At 20:20 5/12/2006, Fabio Silva wrote:

Quem puder dê uma ajuda estou estudando para futuros
concursos:

Considerem-se as funções quadráticas definidas por
y=(a+1)x^2 ­ 2ax ­ (3a +7), variável x e parâmetro a.
Todos os gráficos apresentam uma corda comum. Qual
comprimento da corda? Resp: 4sqrt5.


Considerem-se um triangulo ABC onde a medida do ângulo
A é o dobro da medida de B. A medida do lado a, oposto
ao ângulo A, em função dos lados b e c, é_.
Resp: sqrt (b^2 + bc)

Considere um triangulo ret de hip a, sendo h a altura
relativa a hip e r o raio do circulo inscrito no
triangulo. Inscrevem-se neste triangulo um quadrado de
lados sobre os catetos e vértice na hip, e um outro de
lado sobre a hip e vértices sobre os catetos. A razão
entre as medidas dos lados do primeiro e do segundo
quadrado é:. Resp: a+r sobre a+2r


Vlw amigos matemáticos!!!





Yahoo! Music Unlimited
Access over 1 million songs.
http://music.yahoo.com/unlimited
=
Instruções para entrar na lista, sair da lista e usar a lista em
http://www.mat.puc-rio.br/~nicolau/olimp/obm-l.html
=




=
Instruções para entrar na lista, sair da lista e usar a lista em
http://www.mat.puc-rio.br/~nicolau/olimp/obm-l.html
=


Re: [obm-l] Fatoração

2006-10-16 Por tôpico Carlos Victor


Olá  Bruna ,
Adicione  e  subtraia  os  fatores : x^10 ,x^9 , ... x  .Depois  é só 
agrupar  os  fatores : x^11+x^10+x^9 , -(x^10+x^9+x^8)  e  assim 
por  diante  ; onde   o fator  x^2+x+1  será  comum . Conclusão



x^11 + x^7 + 1 = ( x^2+x+1)(x^9-x^8+x^6-x^4+x^3-x+1) , ok ?

[]´s  Carlos  Victor



At 14:05 14/10/2006, Bruna Carvalho wrote:

Fatorar:

P(x) = x^11 + x^7 + 1.




=
Instruções para entrar na lista, sair da lista e usar a lista em
http://www.mat.puc-rio.br/~nicolau/olimp/obm-l.html
=


Re: [obm-l] trigonometria - O retorno!

2006-10-07 Por tôpico Carlos Victor

Olá  marinho ,
Faça o seguinte :

A/2+B/2+C/2 = 90°  , então  cos( A/2+B/2) = sen(C/2) --  cos(A/2).Cos(B/2) 
- sen(A/2).sen(B/2) = sen(C/2) ;


multiplique os dois  lados por 4sen(A/2).sen(B/2)  , adicione  uma unidade 
ambos os membros ,


 substitua senA =2sen(A/2).cos(B/2) ,senB=2sen(B/2).cos(B/2) 
e   sen(A/2)^2 =(1-cosA)/2 , fazendo o mesmo  para sen(B/2)^2 .


No  final teremos :  senA.senB -+cosA+cosB -cosA.cosB = 1 + 
4sen(A/2).sen(B/2).sen(C/2)  e ,


como senA.senB-cosA.cosB=cosC   chegaremos  ao resultado  pedido , ok ?

[]´s  Carlos  Victor





At 20:05 7/10/2006, Marinho Kamiroski wrote:

Essa é meio q clássica, vem me perseguindo a meses e nada de sair.

cos(a) + cos(b) + cos(c) = 1 + 4*sen(a/2)*sen(b/2)*sen(c/2)
com a + b + c =180° (angulos internos de um triangulo)

_
Descubra aqui como mandar Torpedos Messenger! 
http://www.msn.com.br/artigos/maguire/default.asp 
http://www.msn.com.br/artigos/maguire/default.asp


=
Instruções para entrar na lista, sair da lista e usar a lista em
http://www.mat.puc-rio.br/~nicolau/olimp/obm-l.html
=



=
Instruções para entrar na lista, sair da lista e usar a lista em
http://www.mat.puc-rio.br/~nicolau/olimp/obm-l.html
=


Re: [obm-l] LIMITES

2006-05-21 Por tôpico Carlos Victor


Olá ,
Para o segundo limite temos :
lim(x--+inf) sen(x^1000)/x = lim(
1/x.sen(x^1000) , como sendo uma função
infitesima multiplicada por um limitada ; ou
seja a resposta é zero .
Tem certeza que a questão (1)
esta correta ?
[]´s Carlos Victor

At 10:37 21/5/2006, Klaus Ferraz wrote:
1)Determine lim(n-+inf)
(1+1/2)*(1+1/2^2)*(1+1/2^3)*...*(1+1/2^n).
2)Determine lim(x--+inf) sen(x^1000)/x

Grato.

Yahoo!
Messenger com voz - Instale agora e faça ligações de graça.




Re: [obm-l] Teoria dos Numeros

2006-01-26 Por tôpico Carlos Victor


Olá Klauss ,
(x+1)^3 - x^3 = y^2 , onde 3(2x+1)^2 = (2y-1)(2y+1) .
Observe que podemos concluir que 
:
a) Ou 2y-1 = a^2 e 2y+1 = 3b^2

b) Ou 2y-1 = 3c^2 e 2y+1 =
d^2 .

Observe que 3b^2 = a^2
+2 é a única que pode ocorrer
e, como a é ímpar ,
podemos escrever 
a = 2t +1 e 4y = 2(a^2+1)
implicando y = t^2 + (t+1)^2 ,
ok ?
OBS : (1) Esta questão se encontra no
Livro POWER PLAY de
EDWARD J. BARBEAU da MAA ; inclusive com
a solução acima 
(2) O interessante é que para 3x^2+3x
+1 =y^2 tem para solução geral
:
x1 = 4y+7x+3 e y1 = 7y+12x+6
com x e y conhecidos . Exemplo
: x1 = 104 e y1 =181 ; Lindo não é
?

[]´s Carlos Victor


At 20:23 24/1/2006, Klaus Ferraz wrote:
Mostre que a diferença entre os
cubos de dois numeros inteiros consecutivos é igual ao quadrado de um
inteiro, entao esse inteiro é igual a soma dos quadrados de dois inteiros
consecutivos.
Ex: 8^3-7^3=169. 2^2+3^2=13.

Grato.

Yahoo! doce lar.
Faça
do Yahoo! sua homepage. 



Re: [obm-l] Geometria plana

2006-01-26 Por tôpico Carlos Victor



Olá Vinícius ,
Sejam d1, d2 e d3 as distâncias e L o lado do
triângulo ;escolha um ponto exterior ao
triângulo de tal maneira a construir um
triângulo equilátero de lados iguais a d1, por
exemplo . Utilize a congruência de
triângulos( triângulos de lados L ,d1 e d3) e a 
Lei do co-seno( como o Júnior comentou em
um dos seus e-mails) para
chegar à solução , ok ?
[]´s Carlos Victor

At 17:40 25/1/2006, vinicius aleixo wrote:
Como posso determinar a área de
um triagulo equilátero conhecendo a distancia de um ponto qualquer (P) em
seu interior aos vértices do triângulo(a,b,c)??

Abraços,

Vinícius Meireles Aleixo

Yahoo! doce lar.
Faça
do Yahoo! sua homepage. 



Re: [obm-l] MAXIMO

2006-01-07 Por tôpico Carlos Victor



Olá ,
Use o multiplicador de Lagrange para a
função F = E - k( a+b+c-1=0)  , encontre
a=b=c=1/3 e que o valor máximo
de 
E é igual a 6sqrt(2) , ok ?
[ ]´s Carlos Victor

At 10:36 7/1/2006, Klaus Ferraz wrote:
Calcule o valor maximo da
expressao E=raiz(9a+5)+raiz(9b+5)+raiz(9c+5), a, b e c reais positivos.,
a+b+c=1

Yahoo! doce lar.
Faça
do Yahoo! sua homepage. 



[obm-l] Re: [obm-l] 3 Questõeszinahs de Calculo + 1 de bonus pra se pensar!

2006-01-06 Por tôpico Carlos Victor


Olá,
Para  a primeira  questão  ítem (b)  é  só  lembrar  do seguinte : Seja 
f  uma  função  contínua  em [0,1] , 
lim{ 1/n[f(1/n) + f(2/n) +... f(n/n)]} = integral  de f(x)dx 
com  0 x  1  e  no  problema  a  função
é  
f(x) =  sqrt(1/(1+x)) . Determinando  a integral definida encontraremos 
o resultado  em  questão , ok ?

Para  a Questão  Especial  você  encontrará  uma  solução  elegante
( usando  o princípio de Cavalieri ) no  Livro  de  Olimpíadas 
Brasileiras  de Matemática (  1ª  a  8ª)  da  SBM .  A resposta 
é  16r^3 /3 .

Para  a questão  1a  basta  usar  o mesmo  fato  de  1b (  de outra 
mensagem ) . Para  a  segunda  questão , use  o fato de que  a função  é 
crescente  em [0,1] e  considere o  quadrado  de lado  1  com  um dos
vértices  na origem . observe  que  1/3  é  valor  da  área  sob  a curva
 e , que  a parte  acima da  curva  e dentro  do quadrado é  2/3 , que 
será  o valor da área  sob  a curva quando  considerarmos  a função
 inversa , ok ?( já  que  a função inversa  é simétrica  em relação à 
reta y=x)

[]´s  Carlos  Victor



At 23:13 4/1/2006, you wrote:
Olá
amigos...
Estou com uma lista enorme de exercícios de calculo aqui comigo e fiquei
enrolado em 3 dessas questões...
aí vão elas:






Re: [obm-l] 2 probleminhas

2006-01-04 Por tôpico Carlos Victor

Olá  Fábio ,

Talvez  o enunciado  da primeira questão  seja :

Em  um grupo  de 20  pessoas qual a probabilidade de
que haja pelo menos 2 delas nascidas num mesmo mês  e no  mesmo  dia ?( 
ou  seja , aniversário  no mesmo  dia)


Esta  questão  está  resolvida   no Matemática  Elementar  (  vol 5)  e  no 
Livro  do  Morgado (  Análise  Combinatória   e  probabilidade da  SBM) . 
A  resposta  é


aproximadamente  41% , Ok ?  Verifique

[]´s  Carlos  Victor




At 23:38 3/1/2006, Fabio Silva wrote:

Quem quiser ajudar-me será bem vindo:
1) Em um grupo de 20 pessoas, qual a probabilidade de
que haja pelo menos 2 delas nascidas num mesmo mês?

2) Ao preço de p reais, um fabricante consegue vender,
diariamente, 800-200p pacotes de biscoito. A
fabricação diária de x pacotes de biscoito custa-lhe
100+0,2x reais. Qual é o valor de p para o qual o
lucro do fabricante é máximo?

Desde já agradeço a ajuda dos colegas.



__
Yahoo! DSL ­ Something to write home about.
Just $16.99/mo. or less.
dsl.yahoo.com

=
Instruções para entrar na lista, sair da lista e usar a lista em
http://www.mat.puc-rio.br/~nicolau/olimp/obm-l.html
=




=
Instruções para entrar na lista, sair da lista e usar a lista em
http://www.mat.puc-rio.br/~nicolau/olimp/obm-l.html
=


Re: [obm-l] Usando o Maple

2005-07-24 Por tôpico Carlos Victor


Olá  Marcos,

Esta  substituição   não  resolveu ;  caso  exista  outra me informe , ok ?

[]´s  Carlos Victor

At 01:36 24/7/2005, Marcos Paulo wrote:

Carlos Victor wrote:


Olá  pessoal ,

Gostaria  de saber  como calcular  diretamente  usando o  Maplea 
seguinte  soma  :


 *(5+2*sqrt(13))^(1/3) + (5-2*sqrt(13))^(1/3)

*Agradeço  desde já  qualquer  ajuda


[]´sCarlos  Victor


Tente substituir o 13 por 13.0

[]'s MP
=
Instruções para entrar na lista, sair da lista e usar a lista em
http://www.mat.puc-rio.br/~nicolau/olimp/obm-l.html
=




=
Instruções para entrar na lista, sair da lista e usar a lista em
http://www.mat.puc-rio.br/~nicolau/olimp/obm-l.html
=


[obm-l] Usando o Maple

2005-07-23 Por tôpico Carlos Victor


Olá pessoal ,
Gostaria de saber como calcular diretamente
usando o Maple  a seguinte soma :

(5+2*sqrt(13))^(1/3)
+ (5-2*sqrt(13))^(1/3)
Agradeço desde já
qualquer ajuda

[]´s Carlos Victor





Re: [obm-l] Quest geo plan

2005-07-16 Por tôpico Carlos Victor

Olá   Eduado ,

Trace  a  base  média  MP  e  conclua   que  PN = BN/2 ( 
para  facilitar  chame  BN  de  2x  e CN  será  4x   e consequentemente PC 
será  3x ).Seja  h   a  metade  da  altura  do trapézio ABCD e , 
consequentemente   a altura  do triângulo MPN  será  h/3 . Daí  teremos :


area(MDCP) + area(MPN) = area(AMPB) - area(MPN)e   com  y+ 10 = 2z ; 
onde z = MP   e  AB = y .  Conlua  então  que  y = 20 , ok 
?  confira  as  contas .


[]´s  Carlos  Victor






At 18:18 16/7/2005, e-m-b wrote:

Alguém poderia me ajudar nessa aqui:

Num trapézio ABCD, de bases AB e CD, AB  CD, M é o ponto médio do lado AD;
N está sobre o lado BC e 2*BN = NC. Sabendo-se que as áreas dos
quadriláteros ABNM e CDMN são iguais e que DC = 10, calcule AB.

Essa questão foi da Fuvest 2003. Estou tentando resolvê-la e não consigo
achar saída, embora já tenha feito bastantes questões parecidas com esta.
Qualquer ajuda será bem vinda.
Grato,
Eduardo Beltrão




=
Instruções para entrar na lista, sair da lista e usar a lista em
http://www.mat.puc-rio.br/~nicolau/olimp/obm-l.html
=


RE: [obm-l] Quest geo plan

2005-07-16 Por tôpico Carlos Victor

Olá  Diogo ,

Acredito  que  você  se enganou  nos  seguintes  passos :

1)pelo  que  notei  você   considerou  h   a metade  da  altura  do 
trapézio ABCD , ok ?


2) Área de ABNM=área de ABPM - área de MNP=(AB+PM)*h/2 - área de MNP
Área de CDMN=área de CDMP + área de MNP=(10+PM)h/2 + área de MNP  , ok ?
é  evidente  que  quando  iguala  , PM  desaparece  da igualdade  final .

3)  Em  2h*(AB - 10)=4*[(AB +10)/2]*(h/6)  deveria  ser  2h*(AB - 
10)=4*[(AB +10)/2]*(h/3)  , onde  teremos   AB = 20 . ok?


Confira  as  minhas  contas , ok ?

Mas   de  qualquer  forma  foi  uma  boa  idéia .

[]´s  Carlos  Victor



At 19:28 16/7/2005, Diogo B. Moraes M. de Holanda wrote:

Não sei se está correto, se virem algum erro por favor corrijam:

Considerando P o ponto médio do lado BC e h a altura temos:
Área de ABNM=área de ABPM - área de MNP=AB*h/2 - área de MNP
Área de CDMN=área de CDMP + área de MNP=10h/2 + área de MNP

Como as áreas são iguais, temos:
ABh/2 - área de MNP=10h/2 + área de MNP
h*(AB -10)/2= 2*(área de MNP)
h*(AB - 10)=4*(área de MNP)
h*(AB - 10)=4*(PM*altura referente a PM)/2
2h*(AB - 10)=4*[(AB +10)/2]*(h/6)
2h*(AB - 10)=h*(AB + 10)/3
6h*(AB - 10)=h*(AB +10)
6h*AB - 60h=AB*h + 10h
5AB*h=70h
5AB=70
AB=14


From: e-m-b [EMAIL PROTECTED]
Reply-To: obm-l@mat.puc-rio.br
To: obm-l@mat.puc-rio.br
Subject: [obm-l] Quest geo plan
Date: Sat, 16 Jul 2005 18:18:12 -0300

Alguém poderia me ajudar nessa aqui:

Num trapézio ABCD, de bases AB e CD, AB  CD, M é o ponto médio do lado AD;
N está sobre o lado BC e 2*BN = NC. Sabendo-se que as áreas dos
quadriláteros ABNM e CDMN são iguais e que DC = 10, calcule AB.

Essa questão foi da Fuvest 2003. Estou tentando resolvê-la e não consigo
achar saída, embora já tenha feito bastantes questões parecidas com esta.
Qualquer ajuda será bem vinda.
Grato,
Eduardo Beltrão


_
MSN Messenger: converse online com seus amigos .
http://messenger.msn.com.br

=
Instruções para entrar na lista, sair da lista e usar a lista em
http://www.mat.puc-rio.br/~nicolau/olimp/obm-l.html
=




=
Instruções para entrar na lista, sair da lista e usar a lista em
http://www.mat.puc-rio.br/~nicolau/olimp/obm-l.html
=


Re: [obm-l] QUESTÃO DE GEOMETRIA

2005-07-15 Por tôpico Carlos Victor



Olá Brunno ,
Suponha que os vértices sejam A( do
ãngulo reto) , B e C e, que I e G
sejam os incentro e baricentro, respectivamente .
Seja r o raio do círculo inscrito .
Tomando IG paralelo ao lado AB
e traçando uma perpendicular ao
lado AB de G encontrando
AB em P , teremos :
b = AC = 3r ; já que GP = r .
Como ( b+c-a)/2 = r encontramos b +3c =
3a e elevando ao quadrado esta
igualdade , chegamos a 4b^2 = 3bc
.
Como bc = 1200 encontramos b
=30 e c = 40 , ok ?

[]´s Carlos Victor



At 20:12 14/7/2005, profbrunno
wrote:

Poderia me ajudar com essa questao?



Calcular os lados de um triangulo retangulo que tem a area de 600m²,
sabendo que a reta determinada pelo incentro e baricentro é paralela a um
dos catetos.



Um abraço




[obm-l] Divisor- Novamente-(Fatoração)

2005-06-28 Por tôpico Carlos Victor

Olá   Pessoal ,

Não   tenho  certeza  se  alguém  já  fatorou   a  expressão

K = 2^33-2^19-2^17-1  ;  mas   estive  na  semana  passada  com  o

Antonio Luis  ( Gandhi )  e  ele  me   disse  que   a  solução  é   a 
seguinte :



 K = (2^11)^3 -  (2^6)^3  -1^3  - 3.(2^11).(2^6)e  fazendo   x = 
2^11  ; y = 2^6  e  z = 1


temos   que  x^3 - y^3 - z^3  - 3xyzé  divisível  por  x-y-z = 1983  , ok ?


[]´s   Carlos  Victor



=
Instruções para entrar na lista, sair da lista e usar a lista em
http://www.mat.puc-rio.br/~nicolau/olimp/obm-l.html
=


  1   2   >